Sunteți pe pagina 1din 39

WEEK 2

INSURANCE DOCTRINE:
It is basic that all the provisions of the insurance policy should be examined and interpreted in consonance
E. Essential Elements
with each other. All its parts are reflective of the true intent of the parties. The policy cannot be construed
GULF RESORTS INC vs PHILIPPINE CHARTER INSURANCE CORPORATION (2005) piecemeal. Certain stipulations cannot be segregated and then made to control; neither do particular words
or phrases necessarily determine its character.
FACTS: Gulf Resorts, Inc at Agoo, La Union was insured with American Home Assurance Company
which includes loss or damage to shock to any of the property insured by this Policy occasioned by or Section 2(1) of the Insurance Code defines a contract of insurance as an agreement whereby one undertakes
through or in consequence of earthquake for a consideration to indemnify another against loss, damage or liability arising from an unknown or
contingent event. Thus, an insurance contract exists where the following elements concur:
July 16, 1990: an earthquake struck Central Luzon and Northern Luzon so the properties and 2 swimming 1. The insured has an insurable interest;
pools in its Agoo Playa Resort were damaged 2. The insured is subject to a risk of loss by the happening of the designated peril;
3. The insurer assumes the risk;
August 23, 1990: Gulf's claim was denied on the ground that its insurance policy only afforded earthquake 4. Such assumption of risk is part of a general scheme to distribute actual losses among a large group of
shock coverage to the two swimming pools of the resort persons bearing a similar risk; and
5. In consideration of the insurer's promise, the insured pays a premium.
Petitioner insists that the parties have intended to extend the coverage through the attachment of the phrase An insurance premium is the consideration paid an insurer for undertaking to indemnify the insured against
"Subject to: Other Insurance Clause, Typhoon Endorsement, Earthquake Shock Endorsement, Extended a specified peril. In fire, casualty, and marine insurance, the premium payable becomes a debt as soon as
Coverage Endorsement, FEA Warranty & Annual Payment Agreement on Long Term Policies" to the the risk attaches.
insurance policy.
A contract of adhesion is one wherein a party, usually a corporation, prepares the stipulations in the
ISSUE: Whether or not the insurance policy earthquake shock coverage extends to other property aside contract, while the other party merely affixes his signature or his "adhesion" thereto. Consequently, any
from the two swimming pools. ambiguity therein is resolved against the insurer, or construed liberally in favor of the insured.

HELD: NO. Petitioner cannot focus on the earthquake shock endorsement to the exclusion of the other PHILAMCARE HEALTH SYSTEMS, INC., vs. COURT OF APPEALS and JULITA TRINOS
provisions. All the provisions and riders, taken and interpreted together, indubitably show the intention of G.R. No. 125678 March 18, 2002
the parties to extend earthquake shock coverage to the two swimming pools only. YNARES-SANTIAGO, J.:

A careful examination of the premium recapitulation will show that it is the clear intent of the parties to Facts:ErnaniTrinos, deceased husband of JulitaTrinos, applied for a health care coverage withPhilamcare
extend earthquake shock coverage only to the two swimming pools. Health Systems, Inc. In the standard application form, he answered “NO” to the following question:

In the subject policy, no premium payments were made with regard to earthquake shock coverage, except Have you or any of your family members ever consulted or been treated for high blood pressure, heart
on the two swimming pools. There is no mention of any premium payable for the other resort properties trouble, diabetes, cancer, liver disease, asthma or peptic ulcer? (If Yes, give details).
with regard to earthquake shock. This is consistent with the history of petitioner’s previous insurance
policies from AHAC-AIU. Coverage of the health care agreement (HCA):
approved for a period of one year, Renewed 3 times yearly: March 1, 1988 - March 1, 1990; March 1, 1990
In sum, there is no ambiguity in the terms of the contract and its riders. Petitioner cannot rely on the general – June 1, 1990. The amount of coverage was increased to a maximum sum of P75,000.00 per disability.
rule that insurance contracts are contracts of adhesion which should be liberally construed in favor of the
Ernani’s entitlement under HCA:
insured and strictly against the insurer company which usually prepares it. A contract of adhesion is xxx hospitalization benefits, whether ordinary or emergency, listed therein
out-patient benefits" such as annual physical examinations, preventive health care and other out-patient
We cannot apply the general rule on contracts of adhesion to the case at bar. Petitioner cannot claim it did services.
not know the provisions of the policy. From the inception of the policy, petitioner had required the
respondent to copy verbatim the provisions and terms of its latest insurance policy from AHAC-AIU. Ernaniwas subsequently confined. HISTORY (everything happened within the period of coverage):

Page 1 of 39
Ernani suffered a heart attack and was confined at the Manila Medical Center (MMC) for one month Section 3 of the Insurance Code states that any contingent or unknown event, whether past or future, which
beginning March 9, 1990. may damnify a person having an insurable interest against him, may be insured against. Every person has
Julita tried to claim the benefits under the health care agreement. an insurable interest in the life and health of himself. Section 10 provides:
Philamdenied her claim saying that the Health Care Agreement was void. there was a concealment
regarding Ernani’s medical history. Doctors at the MMC allegedly discovered at the time of Ernani’s Every person has an insurable interest in the life and health:
confinement that he was hypertensive, diabetic and asthmatic, contrary to his answer in the application (1) of himself, of his spouse and of his children;
form. (2) of any person on whom he depends wholly or in part for education or support, or in whom he has a
Julita paid the hospitalization expenses herself, amounting to about P76,000.00 pecuniary interest;
Ernani was discharged at MMC (3) of any person under a legal obligation to him for the payment of money, respecting property or service,
He was attended by a physical therapist at home. of which death or illness might delay or prevent the performance; and
Again he was admitted at the Chinese General Hospital. (4) of any person upon whose life any estate or interest vested in him depends.
Julita brought her husband home again due to financial difficulties. In the case at bar, the insurable interest of respondent’s husband in obtaining the health care agreement was
In the morning of April 13, 1990, Ernani had fever and was feeling very weak. his own health. The health care agreement was in the nature of non-life insurance, which is primarily a
Julita was constrained to bring him back to the Chinese General Hospital where he died on the same day. contract of indemnity. Once the member incurs hospital, medical or any other expense arising from
sickness, injury or other stipulated contingent, the health care provider must pay for the same to the extent
On July 24, 1990, respondent instituted with the Regional Trial Court of Manila, Branch 44, an action for agreed upon under the contract.
damages against Philam and its president, Dr. Benito Reverente, She asked for reimbursement of her
expenses plus moral damages and attorney’s fees. After trial, the lower court ruled against Philam, ordered: NONE, there was no concealment of material facts.
1. Defendants to pay and reimburse the medical and hospital coverage of the late ErnaniTrinos in the Petitioner cannot rely on the stipulation regarding "Invalidation of agreement" which reads:
amount of P76,000.00 plus interest, until the amount is fully paid to plaintiff who paid the same; Failure to disclose or misrepresentation of any material information by the member in the application or
2. Defendants to pay the reduced amount of moral damages of P10,000.00 to plaintiff; medical examination, whether intentional or unintentional, shall automatically invalidate the Agreement
3. Defendants to pay the reduced amount ofP10,000.00 as exemplary damages to plaintiff; from the very beginning and liability of Philamcare shall be limited to return of all Membership Fees paid.
4. Defendants to pay attorney’s fees of P20,000.00, plus costs of suit. An undisclosed or misrepresented information is deemed material if its revelation would have resulted in
the declination of the applicant by Philamcare or the assessment of a higher Membership Fee for the
CA: affirmed the decision of the trial court but deleted all awards for damages and absolved petitioner benefit or benefits applied for.
Reverente.Denied MR.
The answer assailed by petitioner was in response to the question relating to the medical history of the
Issues: applicant. This largely depends on opinion rather than fact, especially coming from respondent’s husband
Whether health care agreements are considered insurance contracts. who was not a medical doctor. Where matters of opinion or judgment are called for, answers made in good
Whether there was concealment of material facts on the part of Ernani that rendered the HCA void by faith and without intent to deceive will not avoid a policy even though they are untrue. Thus,
virtue of the "Invalidation of agreement" contained in the contract.
Suppose there was concealment, what are the steps Philam should have done? (A)lthough false, a representation of the expectation, intention, belief, opinion, or judgment of the insured
will not avoid the policy if there is no actual fraud in inducing the acceptance of the risk, or its acceptance
Ruling: at a lower rate of premium, and this is likewise the rule although the statement is material to the risk, if the
YES, it is an insurance contract. statement is obviously of the foregoing character, since in such case the insurer is not justified in relying
Section 2 (1) of the Insurance Code defines a contract of insurance as an agreement whereby one upon such statement, but is obligated to make further inquiry. There is a clear distinction between such a
undertakes for a consideration to indemnify another against loss, damage or liability arising from an case and one in which the insured is fraudulently and intentionally states to be true, as a matter of
unknown or contingent event. An insurance contract exists where the following elements concur: expectation or belief, that which he then knows, to be actually untrue, or the impossibility of which is
(1) The insured has an insurable interest; shown by the facts within his knowledge, since in such case the intent to deceive the insurer is obvious and
(2) The insured is subject to a risk of loss by the happening of the designated peril; amounts to actual fraud. (Underscoring ours)
(3) The insurer assumes the risk;
(4) Such assumption of risk is part of a general scheme to distribute actual losses among a large group of The fraudulent intent on the part of the insured must be established to warrant rescission of the insurance
persons bearing a similar risk; and contract. Concealment as a defense for the health care provider or insurer to avoid liability is an affirmative
(5) In consideration of the insurer’s promise, the insured pays a premium. defense and the duty to establish such defense by satisfactory and convincing evidence rests upon the
provider or insurer. In any case, with or without the authority to investigate, petitioner is liable for claims
made under the contract. Having assumed a responsibility under the agreement, petitioner is bound to
Page 2 of 39
answer the same to the extent agreed upon. In the end, the liability of the health care provider attaches once INSULAR V EBRADO G.R. NO. L-44059 OCTOBER 28, 1977
the member is hospitalized for the disease or injury covered by the agreement or whenever he avails of the
covered benefits which he has prepaid. Facts:
J. Martin:
Philamshloud have followed Section 27 of the Insurance Code: Cristor Ebrado was issued by The Life Assurance Co., Ltd., a policy for P5,882.00 with a rider
"a concealment entitles the injured party to rescind a contract of insurance." The right to rescind should be for Accidental Death. He designated Carponia T. Ebrado as the revocable beneficiary in his policy. He
exercised previous to the commencement of an action on the contract.In this case, no rescission was made. referred to her as his wife.
Besides, the cancellation of health care agreements as in insurance policies require the concurrence of the Cristor was killed when he was hit by a failing branch of a tree. Insular Life was made liable to pay
following conditions: the coverage in the total amount of P11,745.73, representing the face value of the policy in the amount of
Prior notice of cancellation to insured; P5,882.00 plus the additional benefits for accidental death.
Notice must be based on the occurrence after effective date of the policy of one or more of the grounds Carponia T. Ebrado filed with the insurer a claim for the proceeds as the designated beneficiary therein,
mentioned; although she admited that she and the insured were merely living as husband and wife without the benefit
Must be in writing, mailed or delivered to the insured at the address shown in the policy; of marriage.
Must state the grounds relied upon provided in Section 64 of the Insurance Code and upon request of Pascuala Vda. de Ebrado also filed her claim as the widow of the deceased insured. She asserts that she is
insured, to furnish facts on which cancellation is based. the one entitled to the insurance proceeds.
None of the above pre-conditions was fulfilled in this case. Insular commenced an action for Interpleader before the trial court as to who should be given the proceeds.
The court declared Carponia as disqualified.
Anent the incontestability of the membership of respondent’s husband, we quote with approval the
following findings of the trial court: Issue: WON a common-law wife named as beneficiary in the life insurance policy of a legally married man
can claim the proceeds in case of death of the latter?
(U)nder the title Claim procedures of expenses, the defendant Philamcare Health Systems Inc. had twelve
months from the date of issuance of the Agreement within which to contest the membership of the patient if Held: No. Petition
he had previous ailment of asthma, and six months from the issuance of the agreement if the patient was
sick of diabetes or hypertension. The periods having expired, the defense of concealment or Ratio:
misrepresentation no longer lie. Section 50 of the Insurance Act which provides that "the insurance shall be applied exclusively to the
proper interest of the person in whose name it is made"
The word "interest" highly suggests that the provision refers only to the "insured" and not to
F. Applicability of Civil Code the beneficiary, since a contract of insurance is personal in character. Otherwise, the prohibitory laws
against illicit relationships especially on property and descent will be rendered nugatory, as the same could
Art. 2011. The contract of insurance is governed by special laws. Matters not expressly provided for in easily be circumvented by modes of insurance.
such special laws shall be regulated by this Code. (n) When not otherwise specifically provided for by the Insurance Law, the contract of life insurance is
governed by the general rules of the civil law regulating contracts. And under Article 2012 of the same
Art. 2012. Any person who is forbidden from receiving any donation under Article 739 cannot be named Code, any person who is forbidden from receiving any donation under Article 739 cannot be
beneficiary of a life insurance policy by the person who cannot make any donation to him, according to named beneficiary of a fife insurance policy by the person who cannot make a donation to him. Common-
said article. (n) law spouses are barred from receiving donations from each other.
Article 739 provides that void donations are those made between persons who were guilty of adultery or
Art. 739. The following donations shall be void: concubinage at the time of donation.
There is every reason to hold that the bar in donations between legitimate spouses and those between
(1) Those made between persons who were guilty of adultery or concubinage at the time of the donation; illegitimate ones should be enforced in life insurance policies since the same are based on similar
(2) Those made between persons found guilty of the same criminal offense, in consideration thereof; consideration. So long as marriage remains the threshold of family laws, reason and morality dictate that
(3) Those made to a public officer or his wife, descedants and ascendants, by reason of his office. the impediments imposed upon married couple should likewise be imposed upon extra-marital relationship.
In the case referred to in No. 1, the action for declaration of nullity may be brought by the spouse of the A conviction for adultery or concubinage isn’t required exacted before the disabilities mentioned in Article
donor or donee; and the guilt of the donor and donee may be proved by preponderance of evidence in the 739 may effectuate. The article says that in the case referred to in No. 1, the action for declaration of nullity
same action. (n) may be brought by the spouse of the donor or donee; and the guilty of the donee may be proved by
preponderance of evidence in the same action.

Page 3 of 39
The underscored clause neatly conveys that no criminal conviction for the offense is a condition precedent.
The law plainly states that the guilt of the party may be proved “in the same acting for declaration of nullity (2) Whether or not the complainant was with insurable interest therein when the said policy contract was
of donation.” And, it would be sufficient if evidence preponderates. procured.
The insured was married to Pascuala Ebrado with whom she has six legitimate children. He was also living The complainant has insurable interest in the insured property at the time of the procurement of the
in with his common-law wife with whom he has two children. insurance policy. As the CC provides, “the contract of sale is perfected at the moment there is a meeting of
minds upon the thing which is the object of the contract and upon the price,” and Sec. 15 of the IC allows
ZENITH INSURANCE CORPORATION V. THE INSURANCE COMMISSION- INSURABLE the insurance of a mere contingent or expectant interest in anything if the same is founded on an actual
INTEREST right to the thing, or upon any valid contract.

Facts: As this is the case, mere possession of an equitable title, like that pertaining to the buyer, gives rise to
> Zenith entered into an insurance contract, denominated as Equipment Floater Policy covering a Kato insurable interest in the property in which such title inheres. Furthermore, considering that Zenith’s agent
Bachoe including its accessories and appurtenances thereof, from loss of damage. Complainant paid the had been fully apprised of the circumstances prior to the actual issuance of the policy and the endorsement,
stipulated premiums therefore. it cannot now allege that complainant has no insurable interest on the property insured. Zenith is now
> Within the period of effectivity of the policy, the two pieces of hydraulic wheel gear pumps, which are precluded by the equitable principle of estoppel from impugning and dishonoring the very insurance policy
considered appurtenances and/or parts attached to and/or installed in the Kato BAchoe were lost, stolen contract it issued and the endorsement and increase in the coverage made through its duly authorized agent.
and/or illegally detached by unknown thieves or malefactors
> Despite repeated assurances by Zenith’s soliciting agent, it refused and failed to settle and pay G. Subrogation
complainant’s insurance claim. Art. 2027. No annuity shall be claimed without first proving the existence of the person upon whose life the
> Complainant seeks not only the payment of said insurance claim of 70T plus legal interest, atty’s fees, annuity is constituted.
and litigation expenses, but also the revocation or cancellation of the license of Zenith to do insurance
business. FIREMAN’S FUND INSURANCE COMPANY and FIRESTONE TIRE AND RUBBER COMPANY
> Zenith on the other hand contends that: OF THE PHILIPPINES vs. JAMILA & COMPANY, INC. and FIRST QUEZON CITY
o Complainant is not the real party in interest since the policy carries with it a designated loss payee, the INSURANCE CO., INC
BA Finance Corp
o The policy insures against loss or damage caused by fire and lightning, etc, while theft or robbery is SUMMARY: Jamila supplies security guards to Firestone and assumes their responsibility. When some
NOT insured against in the policy, it not having been expressly mentioned properties of Firestone were lost due to connivance of some security guards, Fireman’s Fund as insurer paid
o Loss nevertheless is excluded under the exception of “infidelity exclusion” by the operator who left it Firestone the value of such and is now subrogated to Firestone’s right to reimbursement. They filed
unguarded, unattended and deserted while entrusted to him, and for failure to give timely notice of loss complaint to recover money when Jamila failed to pay. CFI dismissed complaint as to Jamila citing that
o Complainant and/or BA Finance is guilty of concealment and misrepresentation at the time they secured there is no cause of action as the latter did not consent to subrogation and there are no allegations in the
the policy, because at the time it became operative, the complainant was NOT yet the owner of the property complaint that Firestone investigated the loss. Subsequent MRs, F&F argue that their cause of action is on
insured, the property still hot having been delivered to him, and BA finance had no insurable interest yet, the basis of legal subrogation.
henceforth, the contract of insurance was VOID AB INITIO for lack of insurable interest at the time the SC: There was cause of action on the part of Fireman’s Fund pursuant to Art. 2207. Payment by the assurer
insurance took effect. to the assured operates as an equitable assignment to the assurer of all the remedies which the assured may
have against the third party whose negligence or wrongful act caused the loss.
Issues and Resolutions: DOCTRINE: Loss or injury for risk must be covered by the policy – Under Article 2207, the cause of the
(1) Whether or not the loss through theft or robbery claimed is within the coverage of the policy. loss or injury must be a risk covered by the policy to entitle the insurer to the subrogation. Thus, where the
The Insurance Commissioner, as reiterated by the SC, found for the complainant in this wise: While the insurer pays the insured for a loss which is not a risk covered by the policy, thereby effecting “voluntary
policy enumerated the risks covered, it does NOT, however, in its express terms, limit compensability to payment,” the insurer has no right of subrogation against the third party liable for the loss. Nevertheless,
that stated in the enumeration. The enumerated risks excluded did not include theft or robbery committed the insurer may recover from the third party responsible for the damage to the insured property under
or perpetrated by an unidentified culprit, hence the complainant’s claim for damages is compensable. Article 1236 of the Civil Code.

The foregoing policy is supported by the long time honored doctrine of “contra proferentem: which FACTS:
provides that: “any ambiguity in the policy shall be resolved in favor of the insured and against the Jamila or the Veterans Philippine Scouts Security Agency contracted to supply security guards to Firestone.
insurer”. This is true because insurance contracts are essentially contracts of adhesion and applicants for Jamila assumed responsibility for the acts of its security guards
insurance have no choice but to accept the terms and conditions in the policy even if they are not in full First Quezon City Insurance Co., Inc. executed a bond in the sum of P20k to guarantee Jamila’s obligations
accord therewith. under that contract
Page 4 of 39
May 18, 1963: Properties of Firestone valued at Php 11,925 were lost allegedly due to the acts of its JAMILA: Legal subrogation under Art. 2207 requires the debtor’s consent
employees who connived with Jamila’s security guard o Legal subrogation takes place in the cases mentioned in NCC 1302 and the instant case is not among the
Fireman’s Fund, as insurer, paid to Firestone the amount of the loss and is 3 cases enumerated in that article
now subrogated to Firestone’s right to get reimbursement from Jamila o There could be no subrogation in this case because according to F&F, the contract between Jamila and
Jamila and its surety, First Quezon City Insurance Co., Inc., failed to pay the amount of the loss in spite of Firestone was entered into on June 1, 1965 but the loss complained of occurred on May 18, 1963.
repeated demands.
Fireman’s Fund and Firestone Tire and Rubber Co instituted this complaint against Jamila for the recovery ISSUES:
of the sum of Php 11,925.00 plus interest, damages and attorney’s fees 1) Whether the complaint of Firestone as subrogor (???) states a cause of action against Jamila? (Not really)
Jamila moved to dismiss the complaint on the ground of lack of cause of action 2) Whether the complaint of Fireman’s Fund as subrogee states a cause of action against Jamila? (YES)
(1) complaint did not allege that Firestone, pursuant to the contractual stipulation quoted in the 3) Whether Jamila should reimburse Fireman’s Fund? (Not decided here)
complaint, had investigated the loss and that Jamila was represented in the investigation and
(2) Jamila did not consent to the subrogation of Fireman’s Fund to Firestone’s right to get reimbursement HELD:
from Jamilaand its surety. CFI Decision’s order of dismissal is legally untenable so SET ASIDE with costs against Jamila & Co., Inc.
CFI: Dismissed the complaint as to Jamila on the second ground that there wasno allegation that it RATIO:
had consented to the subrogation and, therefore, Fireman’s Fund had no cause of action against it. [F&F’s counsel gratuitously alleged in their brief that Firestone and Jamila entered into a “contract of guard
Also dismissed the complaint as to First Quezon City Insurance Co., Inc. on the ground of res judicata as services” on June 1, ‘65.That allegation was uncalled for because it is not found in the complaint and so
the same action was previously filed in a civil case which was dismissed because of the failure of the same created confusion which did not exist. No copy of the contract was annexed to the complaint.
plaintiffs and their counsel to appear at the pre-trial. That confusing statement was an obvious error since it was expressly alleged in the complaint that the loss
Firestone and Fireman’s Fund filed MR occurred on May 18, ‘63. The fact that such an error was committed is another instance substantiating the
CFI on F&F’s MR: Set aside its order of dismissal. observation that F&F’s counsel had not exercised due care in the presentation of his case.]
No res judicata as to First Quezon City Insurance Co., because civil case was dismissed without 1) Firestone is really a nominal party in this case as it had already been indemnified for the loss which it
prejudice had sustained. It joined as a party-plaintiff in order to help Fireman’s Fund to recover the amount of the loss
However, due to inadvertence, the lower court did not state in its order of September 3, 1966 why it set from Jamila and First Quezon City Insurance Co., Inc. Firestone had tacitly assigned to Fireman’s Fund its
aside its prior order dismissing the complaint with respect to Jamila. cause of action against Jamila for breach of contract. Sufficient ultimate facts are alleged in the complaint
First Quezon City Insurance Co., Inc. filed its answer to the complaint. to sustain that cause of action.
Jamila, upon noticing that the order had obliterated its victory without any reason therefor, filed MR 2) Fireman’s Fund’s action against Jamila is squarely sanctioned by article 2207. As the insurer, Fireman’s
reconsideration Fund is entitled to go after the person or entity that violated its contractual commitment to answer for the
Invoked the first ground in its original motion to dismiss which had never been passed upon by the lower loss insured against (PAL vs. Heald Lumber Co).
court that complaint did not allege that Firestone, pursuant to the contractual stipulation quoted in CFI erred in applying to this case the rules on novation. F&F in alleging in their complaint that Fireman’s
the complaint, had investigated the loss and that Jamila was represented in the investigation Fund “became a party ininterest in this case by virtue of a subrogation right given in its favor by” Firestone,
CFI on Jamila’s MR: Granted Jamila’s MR. However, it completely ignored the 1st ground but reverted to were not relying on the novation by changeof creditors as contemplated in NCC 1291 and 1300 to 1303
the second ground (no consent to subrogation thus no cause of action). but rather on NCC 2207.
It did not mention Firestone, the co-plaintiff of Fireman’s Fund.
Firestone and Fireman’s Fund filed MR on the ground that Fireman’s Fund Insurance Company was suing Article 2207 is a restatement of a settled principle of American jurisprudence. Subrogation has been
on the basis of legal subrogation whereas CFI erroneously predicated its dismissal order on the theory referred to as the doctrine of substitution. It “is an arm of EQUITY that may guide or even force one to
that there was no conventional subrogation because the debtor’s consent was lacking. pay a debt for which an obligation was incurred but whichwas in whole or in part paid by another” (83
Cited NCC 2207 which provides that “if the plaintiff’s property has been insured, and he has received C.J.S. 576).
indemnity from the insurance company for the injury or loss arising out of the wrong or breach of contract “Subrogation is founded on principles of JUSTICE AND EQUITY, and its operation is governed by
complained of, the insurance company shall be subrogated to the rights of the insured against the principles of equity. It rests onthe principle that substantial justice should be attained regardless of form,
wrongdoer or the person who has violated the contract”. that is, its basis is the doing of complete, essential, andperfect justice between all the parties without regard
CFI on F&F MR: Denied motion to form”(83 C.J.S. 579- 80)
F&F filed 2nd MR and called CFI’s attention to the fact that the issue of subrogation was of no moment Subrogation is a normal incident of indemnity insurance (Aetna L. Ins. Co. vs Moses). Upon payment of
because Firestone, the subrogor (??), is a party-plaintiff and could sue directly Jamila in its own right. the loss, the insurer is entitled to be subrogate pro tanto to any right of action which the insured may
CFI on F&F’S 2nd MR:Denied 2nd MR without resolving contention have against the third person whosenegligence or wrongful act caused the loss (44 Am. Jur. 2nd 745).
Appeal to SC
F&F: CFI’s dismissal of their complaint is contrary to Article 2207 which provides for legal subrogation.
Page 5 of 39
The right of subrogation is of the highest EQUITY. The LOSS IN THE FIRST INSTANCE is that of the WHEREFORE, in view of the foregoing, the present petition is GRANTED. Petitioner’s complaint for
INSURED but AFTERreimbursement or compensation, it becomes the LOSS OF THE INSURER (44 Am. damages against private respondents is reinstated. So the case was remanded to the Trial Court for the trial
Jur. 2d 746). of the merit.
“Although many policies including policies in the standard form, now provide for subrogation, and thus In the pertinent case of Sveriges Angfartygs Assurans Forening v. Qua Chee Gan, supra.,the Court ruled
determine the rights of theinsurer in this respect, the equitable right of subrogation as the legal effect of that the insurer who may have no rights of subrogation due to “voluntary” payment may nevertheless
payment inures to the insurer without any formalassignment or any express stipulation to that effect recover from the third party responsible for the damage to the insured property under Article 1236 of the
in the policy” (44 Am. Jur. 2nd 746). Civil Code.
In the pertinent case of Sveriges Angfartygs Assurans Forening v. Qua Chee Gan, supra., the Court
Stated otherwise, when the insurance company pays for the loss, such payment operates as an equitable ruled that the insurer who may have no rights of subrogation due to “voluntary” payment may
assignment to the insurer of the property and all remedies which the insured may have for the nevertheless recover from the third party responsible for the damage to the insured property under
recovery thereof. That right is not dependent upon, nor does it grow out of, any privity of contract, or upon Article 1236 of the Civil Code.
written assignment of claim, and payment to the insured makes the insurer an assignee in equity (Shambley
v. Jobe-Blackley Plumbing and Heating Co).3) Whether the plaintiffs would be able to prove their cause of SVERIGES ANFARTYGS ASSURANCE VS QUA CHEE GAN (full case)
action against Jamila is another question.
On August 23 and 24, 1947, defendant Qua Chee Gan, a sole proprietorship, shipped on board the S.S.
PAN MALAYAN INSURANCE CORPORATION, petitioner, vs. COURT OF APPEALS, ERLINDA
FABIE AND HER UNKNOWN DRIVER, NAGARA as per bills of lading Exhs. A and B 2,032,000 kilos of bulk copra at Siain, Quezon, consigned to
DAL International Trading Co., in Gdynia, Poland. The vessel first called at the port of Karlshamn,
Sweden, where it unloaded 696,419 kilos of bulk copra. Then, it proceeded to Gdynia where it unloaded
FACTS: the remaining copra shipment. The actual outturn weights in the latter port showed that only 1,569,429
1. Petitioner Panmalay was an insurer of the car of CANLUBANG AUTOMOTIVE RESOURCE CORP. kilos were discharged.
which was bumpt and damaged by the private respondent through its negligent driver.
2. Petitioner PANMALAy paid the amount of insurance to the insured. Because of the alleged confirmed cargo shortage, the Polish cargo insurers had to indemnify the consignee
3. Subrogated on the rights of the insured, petitioner demand payment from the private respondent who for the value thereof. Thereafter, the former sued the ship-owner, the Swedish East Asia
refused to pay the claim of the petitioner. Company, in Gothenburg, Sweden. The latter, in turn, sued defendant and had it summoned to
4. Petitioner filed a complaint against private respondent before the RTC. Gothenburg. Defendant However refused to submit to that court's jurisdiction and its objection was
5. Private respondent filed a motion to dismiss arguing that payment under the “own damage” clause of the sustained.
insurance policy precluded subrogation under Article 2207 of the Civil Code, since indemnification
thereunder was made on the assumption that there was no wrongdoer or no third party at fault. In March, 1951, a settlement was effected between the Polish cargo insurers and the ship
6. The RTC dismissed the complaint aswell as the motion for reconsideration and this was affirmed by the owner. Plaintiff, as the indemnity insurer for the latter, paid approximately $60,733.53 to the Polish
CA. insurers. On August 16, 1954, claiming to have been subrogated to the rights of the carrier, plaintiff sued
defendant before the Court of First Instance of Manila to recover U.S. $60,733.53 plus 17% exchange tax,
ISSUE: with legal interest, as the value of the alleged cargo short shipment and P10,000 as attorney's
WHETHER OR NOT, THE PETITIONER IS ALLOWED TO RECOVERED THE AMOUNT OF fees. Defendant answered in due time and countered with a P15,000 counterclaim for attorney's fees.
INSURANCE IT HAD PAID TO THE INSURED FROM PRIVATE RESPONDENT.
On August 1, 1955, defendant filed a motion to dismiss on the ground of prescription under the
RULING: Carriage of Goods by Sea Act. The lowercourt sustained the motion and plaintiff appealed here. We
According to the Supreme Court, Art. 2207 of the Civil Code states that, “If the plaintiffs property has been reversed the order of dismissals and remanded the case for further proceedings. [1]
insured, and he has received indemnity from the insurance company for the injury or loss arising out of the
wrong or breach of contract complained of, the insurance company shall be subrogated to the rights of the After trial the lower court September 28, 1963, rendered its decisions dismissing the complaint and
insured against the wrongdoer or the person who has violated the contract.” awarding P10,000 as attorney's fees to defendant. It ruled (a) that there was no shortshipment on
This was founded on the well-settled principle of subrogation. If the insured property is destroyed or defendant's part (b) that plaintiff's insurance policy did not cover the shortshipment and (c)
damaged through the fault or negligence of a party other than the assured, the insurer, upon payment to the defendant was merely acting as an agent of Louis Dreyfus & Co., who was the real shipper.
assured, will be subrogated to the rights of the assured to recover from the wrongdoer to the extent that the
insurer has been obligated to pay. Payment by the insurer to the assured operates as an equitable assignment Taking issue with all the foregoing, plaintiff has interposed the present appeal to Us on questions of
to the former of all remedies which the latter may have against the third party whose negligence or fact and law, the amount involved exceeding P200,000.00.
wrongful act caused the loss.
Page 6 of 39
Issue: recover, and not bank on the weakness of the defense. Plaintiff here failed to establish its case by
preponderance on evidence.
Was the non-presentation of the insurance policy fatal to plaintiff's case? The lower court ruled so,
reasoning that unless the same as the best evidence were presented, it could not be conclusively determined On the question whether defendant is the real shipped or merely an agent of Louis Dreyfus & Co., suffice it
if "liability for short shipment" was a covered risk. And the rule is that an insurer who pays the insured for to say that although on Exhibit A and B his name appears as the shipper, yet the very loading certificate,
loss or liability not covered by the policy is not subrogated to the latter.[2] However, even assuming that Exhibit 3 [5-Deposition of Horle], issued and signed by the Chief Mate, and Master of the S.S. NAGARA
there was unwarranted - or "volunteer" - payment, plaintiff could still recover what it paid - in effect - to the shows that defendant was acting merely for account of Louis Dreyfus & Co. The other documentary
carrier from defendants shipper under Art. 1236 of the Civil Code which allows a third person who pays on exhibits[7] confirm this. Anyway, in whatever capacity defendant is considered, it cannot be liable since
behalf of another to recover from the latter, although there is no subrogation. But since the payment here no shortshipment was shown.
was without the knowledge and consent of defendant, plaintiff's right of recovery is defeasible by
the former's defenses since the Code is clear that the recovery only up to the amount by which the Plaintiff's action against defendant cannot, however, be considered as clearly unfounded as to warrant an
defendant was benefited. award of attorney's fees as damages to defendant under par. 4, Art. 2208 of the Civil Code. The facts do not
show that plaintiff's cause of action was so frivolous or untenably as to amount to gross and evident bad
This brings Us to the crux of them case: Was there a shortshipment? To support its case, plaintiff theorizes faith.[8]
that defendant had two shipments at Siain, Quezon province: (1) 812,800 kilos for Karlshamn and (2)
2,032,000 kilos for Gdynia. The Karlshamn shipment was asserted to have been covered by a separate bill WHEREFORE, but for the award of attorney's fees to defendant which is eliminated, the decision
of lading which however was allegedly lost subsequently. Thus, the 696,419 kilos of copra unloaded appealed from is, in all other respects, hereby affirmed. Costs against plaintiff-appellant.
in Karlshamn was not part of the Gdynia shipment and cannot explain the confirmed shortage at the latter
port. RIZAL SURETY AND INSURANCE CO. vs. MANILA RAILROAD CO. AND MANILA PORT
SERVICE
Plaintiff's cause of action suffers from several fatal defects and inconsistencies. The alleged shipment of
812,800 kilos for Karlshamn is contradicted by plaintiff's admission in paragraphs 2 and 3 of its complaint
Facts:
that defendant shipped only 2,032,000 kilos copra at Siain, purportedly for both Gdynia and Karlshamn. On Nov 29, 1960, a vessel named SS Flying Trader, loaded on board a cargo which is an offset press
[3] Needless to state, plaintiff is bound by such judicial admission. [4] Moreover, the alleged existence of
machine, from Italy to Manila. Upon reaching the port of destination and upon unloading it, it was dropped
the Karlshamn bills of lading is negative by the fact that Exhibits A and B - the bills of lading presented by b the crane which resulted to damages of the machine. The plaintiff as the insurer had paid the consignee,
plaintiff - show that the 2,032,000 kilos of copra loaded in Siain were for Gdynia only. Further destroying Suter, Inc. the amount of P16.5k for the machine and P180.70 for the International Adjustment Bureau as
its case is the testimony of plaintiff's own witness, Mr. Claro Pasicolan, who on direct examination adjuster’s fee. However, the arrastre charges in this particular shipment was paid on the weight or
affirmed[5] that these two exhibits constituted the complete set of documents which them shipping agent in measurement basis whichever is higher, and not on the value thereof.
charge of the vessel S.S. NAGARA issued covering the copra cargo loaded at Slain. In view of
this admission and for want of evidentiary support, plaintiff's belated claim that there is another complete Issue:
set of documents can not be seriously taken. Can the insurance get an amount greater than what was declared?

Lastly, if there really was a separate bill of lading for the Karlshamn shipment, plaintiff could not have Held:
failed to present a copy thereof. Mr. Pasicolan testified[6] that the shipping agent makes 20 copies of the Plaintiff Insurance Company cannot recover from defendants an amount greater than that to which the
documents of which three signed ones are given to the shipper and the rest, marked as non-negotiable bills consignee could lawfully lay claim. The management contract is clear, the amount is limited to P500.
of lading - like Exhibits A and B - are kept on its file. For the three signed copies to be lost,We may If the plaintiff’s property has been insured, and he has received indemnity from the insurance company for
believe, but not for all the remaining 17 other copies. Under the circumstances, it his more reasonable to the injury or loss arising out of the wrong or breach of contract complained of, the insurance company shall
hold that there was no separate shipment intended for Karlshamn, Sweden. be subrogated to the right of the insured against the wrong-doer or the person who has violated the contract.
If the amount paid by the insurance company doer not fully cover the injury or loss, the aggrieved party
As a corollary to the foregoing conclusion, it stands to reason that the copra unloaded in Karlshamn formed shall be entitled to recover the deficiency from the person causing the loss or injury.
part of the same - and only - shipment of defendant intended for Gdynia. Now the fact that the sum total of The insurance have no greater right than the party in interest thereof.
the cargo unloaded at Karlshamn and Gdynia would exceed what appears to have been loaded at Sian by as
much as 233,848 kilos can only show that defendant really overshipped, not shortshipped. And while this PAUL FIRE & MARINE INSURANCE v MACONDRAY & CO
would not tally with defendant's claim of having weighed the copra cargo 100% at Siain, thus exposing a
flaw in defendant's case, yet it is elementary that plaintiff must rely on the strength of it own case to Facts:
Page 7 of 39
-Winthrop Products, Inc., of New York shipped aboard the SS “Tai Ping”, owned and operated by Wilhelm -The stipulation in the bill of lading limiting the common carrier’s liability to the value of the goods
Wilhelmsen218 cartons and drums of drugs and medicine with Winthrop-Stearns Inc., Manila, Philippines appearing in thebill, unless the shipper or owner declares a greater value, is valid and binding.
as consingee. BarberSteamship Lines, Inc., agent of Wilhelm Wilhelmsen issued Bill of Lading No. 34, in -This limitation of the carrier’s liability is sanctioned by the freedom of the contracting parties to establish
the name of Winthrop Products. suchstipulations, clauses, terms, or conditions as they may deem convenient, provided they are not contrary
-The shipment was insured by the shipper against loss and/or damage with the St. Paul Fire & Marine to law,morals, good customs and public policy.
InsuranceCompany. -A stipulation fixing or limiting the sum that may be recovered from the carrier on the loss or deterioration
-“Tai Ping” arrived at the Port of Manila. of the goods is valid, provided it is:
-The said shipment was discharged complete and in good order with the exception of one (1) drum and (a) reasonable and just under the circumstances, and
several cartonswhich were in bad condition. (b) has been fairly and freely agreed upon.
-Because consignee failed to receive the whole shipment and as several cartons of medicine were received -In the case at bar, the liabilities of the defendants- appellees with respect to the lost or damaged shipments
in badorder condition, Winthrop-Sterns Philippines filed the corresponding claim in the amount of areexpressly limited to the C.I.F. value of the goods as per contract of sea carriage embodied in the bill of
Pl,109.67 representingthe C.I.F. value of the damaged drum and cartons of medicine with the carrier lading, whichreads:
and the arrestre. o Whenever the value of the goods is less than $500 per package or other freight unit, their value in the
-However, both refused to pay. calculation and adjustment of claims for which the Carrier may be liable shall for the purpose of avoiding
-Winthrop-Sterns Philippines filed its claim with the insurer, St. Paul Fire & Marine insurance. uncertainties and difficulties in fixing value be deemed to be the invoice value, plus freight and insurance
-The insurance company, on the basis of such claim, paid to the consignee the insured value of the lost and if paid, irrespective of whether any other value is greater or less.
damagedgoods, including other expenses in connection therewith, in the total amount of $1,134.46.
-As subrogee of the rights of the shipper and/or consignee, the insurer, St. Paul Fire & Marine Insurance NATIONAL UNION FIRE INSURANCE VS. STOLT NIELSEN
Co., instituted with the Court of First Instance the present action against the defendants for the recovery of
said amount of $1,134.46, plus costs. EMERGENCY RECIT: United Coconut Chemicals (SHIPPER) shipped distilled fatty acid on board MT
-The Lower court rendered judgment ordering defendants Macondray & Co., Inc., Barber Steamship Lines, “StoltSceptre” (CARRIER). The shipment was insured under a marine cargo policy with National Union
Inc. andWilhelm Wilhelmsen to pay to the plaintiff P300.00. It also held defendants Manila Railroad Fire Insurance Co (INSURER). Upon receipt of the cargo by the consignee in Netherlands, it was totally
Company and Manila PortService to pay to plaintiff, jointly and severally, the sum of P809.67. contaminated. Hence, claim was made on the INSURER of the cargo. The INSURER as subrogee filed a
-The Insurer, , contending that it should recover the amount of $1,134.46 or its equivalent in pesos (the rate claim for damages against the CARRIER with RTC Manila. The CARRIER invoked that arbitration must
of P3.90,instead of P2.00, for every US$1.00), filed a motion for reconsideration, but this was denied. be done pursuant to the Charter. The INSURER opposed, arguing that the provision on arbitration was not
-The Insurer argues that, as subrogee of the consignee, it should be entitled to recover from the defendants- included in the Bill of Lading. SC: The INSURER cannot avoid the binding effect of the arbitration
appelleesthe amount of $1,134.46 which it actually paid to the consignee and which represents the value of clause. By subrogation, it became privy to the Charter Party as fully as the SHIPPER before the latter was
the lost anddamaged shipment as well as other legitimate expenses such as the duties and cost of survey of
indemnified, because as subrogee it stepped into the shoes of the SHIPPER and is subrogated merely to the
said shipment, andthat the exchange rate on the date of the judgment, which was P3.90 for every US$1.00.
latter's rights.
-Defendants-appellees countered that:
o Their liability is limited to the C.I.F. value of the goods, pursuant to contract of sea carriage embodied in FACTS:
the bill of lading that the consignee’s (Winthrop-Stearns Inc.) claim against the carrier (Macondray & Co.,
Inc.,Barber Steamship Lines, Inc., Wilhelm Wilhelmsen and the arrastre operators (Manila Port Service and  On 9 January 1985, United Coconut Chemicals, Inc. shipped 404.774 metric tons of distilled C6-
Manila Railroad Company) was only for the sum of Pl,109.67 C18 fatty acid on board MT "Stolt Sceptre," a tanker owned by Stolt-Nielsen Philippines Inc.,
from Bauan, Batangas, Philippines, consigned to "Nieuwe Matex" at Rotterdam, Netherlands,
ISSUE(S): covered by Tanker Bill of Lading BL No. BAT-1.
1.Whether or not, in case of loss or damage, the liability of the carrier to the consignee is limited to the
C.I.F value of the goods which were lost or damaged  The shipment was insured under a marine cargo policy with Petitioner National Union Fire
2.Whether the insurer who has paid the claim in dollars to the consignee should be reimbursed in its peso Insurance Company of Pittsburg (hereinafter referred to as INSURER), a non-life American
equivalent on the date of discharge of the cargo or on the date of the decision. insurance corporation, through its settling agent in the Philippines, the American International
Underwriters (Philippines), Inc., the other petitioner herein.
HELD:
The appeal is without merit and the judgement of the lower court is affirmed.  Upon receipt of the cargo by the consignee in the Netherlands, it was found to be discoloured and
-The purpose of the bill of lading is to provide for the rights and liabilities of the parties in reference to the totally contaminated. Hence, a claim was made on the Insurer of the cargo. The insurer as
contract tocarry. subrogee filed a claim for damages against the carrier with the RTC of Manila.

Page 8 of 39
 The carrier filed a motion to dismiss on the ground that the case was arbritrable and pursuant to CEBU SHIPYARD V WILLIAM LINES
the charter party as embodied in the bill of lading, arbitration must be done. The insurer opposed
the motion by arguing that the provision on arbitration was not included in the bill of lading and Nature of Case: Petition for Review on Certiorari
even if it was included, it was nevertheless unjust and unreasonable. Digest maker: Erika Potian
 The RTC denied the motion but upon reconsideration, the resolution on the motion to dismiss was SUMMARY: William Lines insured its M/V Manila City under Prudential
suspended or deferred. Guarantee for hull and machinery. Policy contained clause providing that
loss/damage caused by negligence of charterers or repairers are excluded from
 The carrier then filed a petition for review on certiorari with preliminary injunction/TRO which coverage. William Lines brought the vessel to Cebu Shipyard for annual dry-
was granted by the CA. docking and repair. The two executed contracts stipulating the liabilities of both
parties, including that the insurance on the vessel should be maintained by the
ISSUE: Are the terms of the Charter Party, particularly the provision on arbitration, binding on the
owner during period of the contract. After the vessel was transferred to the
INSURER?
docking quay, it caught fire and sank, resulting to its eventual total loss. Cebu
HELD: Yes. The pertinent portion of the Bill of Lading in issue provides in part: Shipyard claims that it is a co-assured under the Marine Hull Insurance Policy
by virtue of Clause 20, and therefore no subrogation can be made by
xxx [A]ll the terms whatsoever of the said Charter except the rate and payment of freight specified Prudential. SC held that it is not and the petition was denied.
therein apply to and govern the rights of the parties concerned in this shipment.xxx DOCTRINE: Intention of parties to make each other co-assured is to be
gleaned from the insurance policy itself and not from any other contract
The provision on arbitration in the Charter Party reads: because the policy denominates the assured and the beneficiaries.
4. Arbitration. Any dispute arising from the making, performance or termination of this Charter FACTS:
Party shall be settled in New York, Owner and Charterer each appointing an arbitrator, who shall
be a merchant, broker or individual experienced in the shipping business; the two thus chosen, if  Cebu Shipyard and Engineering Works, Inc. (CSEW) is engaged in the business of dry-docking
they cannot agree, shall nominate a third arbitrator who shall be an admiralty lawyer. Such and repairing of marine vessels while the Prudential Guarantee and Assurance, Inc. (Prudential) is
arbitration shall be conducted in conformity with the provisions and procedure of the United States in the non-life insurance business.
arbitration act, and a judgment of the court shall be entered upon any award made by said  William Lines, Inc. is in the shipping business. It was the owner of M/V Manila City, a luxury
arbitrator. Nothing in this clause shall be deemed to waive Owner's right to lien on the cargo for passenger-cargo vessel, which caught fire and sank.
freight, deed of freight, or demurrage.  At the time of the unfortunate occurrence sued upon, subject vessel was insured with Prudential
for P45M for hull and machinery. The Hull Policy included an “Additional Perils
Clearly, the Bill of Lading incorporates by reference the terms of the Charter Party. It is settled law that the (INCHMAREE)” Clause covering loss of or damage to the vessel through the negligence of,
charter may be made part of the contract under which the goods are carried by an appropriate reference in among others, ship repairmen
the Bill of Lading. As the respondent Appellate Court found, the INSURER "cannot feign ignorance of the  Petitioner CSEW was also insured by Prudential for third party liability under a Shiprepairer’s
arbitration clause since it was already charged with notice of the existence of the charter party due to an Legal Liability Insurance Policy. The policy was for P10 million only, under the limited liability
appropriate reference thereof in the bill of lading and, by the exercise of ordinary diligence, it could have clause
easily obtained a copy thereof either from the shipper or the charterer.  On Feb. 5, 1991, William Lines, Inc. brought its vessel, M/V Manila City, to the Cebu Shipyard in
Lapulapu City for annual dry-docking and repair.
We hold, therefore, that the INSURER cannot avoid the binding effect of the arbitration clause. By  On Feb. 6, 1991, an arrival conference was held between representatives of William Lines, Inc.
subrogation, it became privy to the Charter Party as fully as the SHIPPER before the latter was and CSEW to discuss the work to be undertaken on the M/V Manila City. The contracts,
indemnified, because as subrogee it stepped into the shoes of the SHIPPER-ASSURED and is subrogated denominated as Work Orders, were signed thereafter., with the following stipulations:
merely to the latter's rights. It can recover only the amount that is recoverable by the assured. And since the o 10. The Contractor shall replace at its own work and at its own cost any work or material
right of action of the SHIPPER-ASSURED is governed by the provisions of the Bill of Lading, which
which can be shown to be defective and which is communicated in writing
includes by reference the terms of the Charter Party, necessarily, a suit by the INSURER is subject to the
same agreements. It has not been shown that the arbitral clause in question is null and void, inoperative, or o 20. The insurance on the vessel should be maintained by the customer and/or owner of
incapable of being performed. Nor has any conflict been pointed out between the Charter Party and the Bill the vessel during the period the contract is in effect.
of Lading.
Page 9 of 39
o The total liability of the Contractor to the Customer or of any sub-contractor shall be 3. WON the doctrine of res ipsa loquitur applies against the crew- YES
limited in respect of any defect or event to the sum of 1M. a. For the doctrine of res ipsa loquitur to apply to a given situation, the following conditions
must concur: (1) the accident was of a kind which does not ordinarily occur unless
 While the M/V Manila City was undergoing dry-docking and repairs within the premises of someone is negligent; and (2) that the instrumentality or agency which caused the injury
CSEW, the master, officers and crew of M/V Manila City stayed in the vessel, using their cabins as was under the exclusive control of the person charged with negligence.
living quarters. Other employees hired by William Lines to do repairs and maintenance work on b. The facts and evidence reveal the presence of these conditions. First, the fire would not
the vessel were also present during the dry-docking. have happened in the ordinary course of things if reasonable care and diligence had been
exercised.
 On February 16, 1991, after subject vessel was transferred to the docking quay, it caught fire and 4. WON the provisions limiting CSEW’s liability for negligence to a maximum of Php 1 million are
sank, resulting to its eventual total loss valid- NO
a. Although contracts of adhesion have been consistently upheld as valid, reliance on such
 On February 21, 1991, William Lines, Inc. filed a complaint for damages against CSEW, alleging
contracts cannot be favored especially where the facts and circumstances warrant that
that the fire which broke out in M/V Manila City was caused by CSEWs negligence and lack of
subject stipulations be disregarded. Tthe facts and circumstances vis-a-vis the nature of
care.
the provision sought to be enforced should be considered, bearing in mind the principles
of equity and fair play.
 Prudential was impleaded as co-plaintiff, after it paid William Lines, Inc. the value of the hull and
machinery insurance on the M/V Manila City. As a result of such payment Prudential was RULING: Petition denied.
subrogated to the claim of P45 million, representing the value of the said insurance it paid.
MANILA MAHOGANY V CA
 Trial Court: CSEW to pay William Lines and Prudential (45M)
FACTS:
 CA: Affirmed TC. Ordered the partial dismissal of the case insofar as CSEW and William Lines
were concerned.
Petitioner insured its Mercedes Benz 4-door sedan with respondent insurance company . The insured
 CSEW claims that the insurance policy does not cover loss resulting from the fault of negligent vehicle was bumped and damaged by a truck owned by San Miguel Corporation (SMC). For the damage
charterers that are assured in the same policy and by virtue of clause 20, it is deemed a co-assured. caused, respondent company paid petitioner ₱ 5,000.00 in amicable settlement. Petitioner’s general
manager executed a Release of Claim, subrogating respondent company to all its right to action against San
ISSUE/S & RATIO: Miguel Corp. Respondent company wrote the Insurer Adjusters, Inc. to demand reimbursements from San
Miguel Corporation of the amount it had paid petitioner. Insurer Adjusters, Inc. refused reimbursement
1. WON CSEW is co-assured, thus losses caused by it are not covered by the policy- NO
alleging that SMC had already paid petitioner ₱ 4,500.00 for the damages to petitioner’s motor vehicle, as
a. The fact that clause 20 benefited petitioner, does not automatically make it a co-assured
evidenced by a cash voucher and Release of Claim executed by the General Manager of petitioner
of William Lines.
discharging SMC from “ all actions, claims, demands the right of action that now exist or hereafter develop
b. Intention of parties to make each other co-assured is to be gleaned from the insurance
arising out of or as a consequence of the accident.
policy itself and not from any other contract because the policy denominates the assured
and the beneficiaries.
c. Prudential named only William Lines, Inc. as the assured. There was no manifestation of Respondent demanded the ₱ 4,500.00 amount from petitioner. Petitioner refused. Suit was filed for
any intention of William Lines Inc to make CSEW a co-assured. When the terms of a recovery. City Court ordered petitioner to pay respondent. CFI affirmed. CA affirmed with modification
contract are clear, its stipulations control. that petitioner was to pay respondent the total amount of ₱ 5,000.00 it had received from respondent.
d. If CSEW were deemed co-assured, it would nullify any claim of William Lines Inc. No
shipowner would agree to make shiprepairer a co-assured because any claim it has under Petitioner’s argument: Since the total damages were valued at P9,486.43 and only ₱ 5,000.00 was received
the policy would be invalidated. Such result could not have been intended by William by petitioner from respondent, petitioner argues that it was entitled to go after SMC to claim the additional
Lines Inc. which was eventually paid to it.
2. WON CSEW had “management and supervisory control“ of the ship at the time the fire broke out-
YES Respondent’s argument: No qualification to its right of subrogation.
a. The factual findings by the CA are conclusive on the parties and are not reviewable by
this Court. ISSUE:
Page 10 of 39
Whether or not the insured should pay the insurer despite that the subrogation in the Release of Claim was Whether or not the insurer should exercise the rights of the insured to which it had been subrogated lies
conditioned on recovery of the total amount of damages that the insured has sustained. solely within the former’s sound discretion. Since the insurer is not a party to the case, its identity is not of
record and no claim is made on its behalf, the private respondent’s insurer has to claim his right to
RULING: reimbursement of the P35,000.00 paid to the insured.

NO. Supreme Court said there being no other evidence to support its allegation that a gentleman’s
agreement existed between the parties, not embodied in the Release of Claim, such Release of Claim must WEEK 3
be taken as the best evidence of the intent and purpose of the parties. CA was correct in holding petitioner
should reimburse respondent ₱ 5,000.00. Dela Cruz vs. Capital Insurance & Surety Co.

When Manila Mahogany executed another release claim discharging SMC from all rights of action after the FACTS: 1.
insurer had paid the proceeds of the policy – the compromise agreement of ₱ 5,000.00– the insurer is Eduardo de la Cruz, employed as a mucker in the Itogon-Suyoc Mines, Inc. in Baguio, was the holder of an
entitled to recover from the insured the amount of insurance money paid. Petitioner by its own acts released accident insurance policy (No. ITO-BFE-170) underwritten by the Capital Insurance & Surety Co., Inc., for
SMC, thereby defeating respondent’s right of subrogation, the right of action against the insurer was also the period beginning November 13, 1956 to November 12, 1957.
nullified. 2. On January 1, 1957, the Itogon-Suyoc Mines, Inc. sponsored a boxing contest for general entertainment.
3. Insured Eduardo de la Cruz, a non-professional boxer participated.
Since the insurer can be subrogated to only such rights as the insured may have, should the insured, after 4.In the course of his bout with another person, likewise a non-professional, of the same height, weight,
receiving payment from the insurer, release the wrongdoer who caused the loss, the insurer losses his rights and size, Eduardo slipped and was hit by his opponent on the left part of the back of the head, causing
against the latter. But in such a case, the insurer will be entitled to recover from the insured whatever it has Eduardo to fall, with his head hitting the rope of the ring.
paid to the latter, unless the release was made with the consent of the insurer. 5. He was brought to the Baguio General Hospital the following day. The cause of death was reported as
hemorrhage, intracranial, left.
6. Simon de la Cruz, the father of the insured and who was named beneficiary under the policy, filed a
FF CRUZ AND CO vs. CA claim with the insurance company for payment of the indemnity under the insurance policy.
7. As the claim was denied, De la Cruz instituted the action in the Court of First Instance of Pangasinan for
specific performance.
Facts:
8. Defendant insurer set up the defense that the death of the insured, caused by his participation in a boxing
A fire broke up from the furniture shop of the petitioner in Caloocan city early September 6, 1974. Prior to
contest, was not accidental and, therefore, not covered by insurance.
that, neighbor of the said shop requested that the petitioner should build a firewall but failed to do so. The
9. After due hearing the court rendered the decision in favor of the plaintiff which is the subject of the
cause of the fire was never discovered. Private respondent got P35k from the insurance on their house and
present appeal.
contents thereof.
10. Eduardo was insured "against death or disability caused by accidental means".
11. Appellant insurer now contends that while the death of the insured was due to head injury, said injury
Issue:
was sustained because of his voluntary participation in the contest.
Whether or not the 35k be deducted from the damages thereof
12. It is claimed that the participation in the boxing contest was the "means" that produced the injury
which, in turn, caused the death of the insured. And, since his inclusion in the boxing card was voluntary on
Ruling:
the part of the insured, he cannot be considered to have met his death by "accidental means".
Since P35k had already been claimed by the respondents, the court held that such amount should be
ISSUE: 1. WON the death of Eduardo de la Cruz is covered by accident insurance policy.
deducted from the award of damages in accordance with Art 2207 NCC
HELD: 1.Yes. While the participation of the insured in the boxing contest is voluntary, the injury was
Art. 2207. If the plaintiff’s property has been insured, and he has received indemnity from the insurance
sustained when he slid, giving occasion to the infliction by his opponent of the blow that threw him to the
company for the injury or loss arising out of the wrong or breach of contract complained of, the insurance
ropes of the ring.
company shall be subrogated to the rights of the insured against the wrongdoer or the person who has
RATIO: 1. The terms "accident" and "accidental", as used in insurance contracts, have not acquired any
violated the contract. If the amount paid by the insurance company does not fully cover the injury or loss,
technical meaning, and are construed by the courts in their ordinary and common acceptation. Thus, the
the aggrieved party shall be entitled to recover the deficiency from the person causing the loss or injury.
terms have been taken to mean that which happen by chance or fortuitously, without intention and design,
Having been indemnified by their insurer, private respondents are entitled only to recover the deficiency
and which is unexpected, unusual, and unforeseen. An accident is an event that takes place without one's
from the petitioner.
foresight or expectation

Page 11 of 39
an event that proceeds from an unknown cause, or is an unusual effect of a known cause and, therefore, not
expected. Ty v. Filipinas Compañia de Seguros - Insurance Policy
2. The generally accepted rule is that, death or injury does not result from accident or accidental means 17 SCRA 364
within the terms of an accident-policy if it is the natural result of the insured's voluntary act, Facts:
unaccompanied by anything unforeseen except the death or injury. > Ty was employed as a mechanic operator by Braodway Cotton Factory at Grace Park, Caloocan.
3. Death or disablement resulting from engagement in boxing contests was not declared outside of the > In 1953, he took personal accident policies from 7 insurance companies (6 defendants), on different
protection of the insurance contract. Failure of the defendant insurance company to include death resulting dates, effective for 12 mos.
from a boxing match or other sports among the prohibitive risks leads inevitably to the conclusion that it > On Dec. 24. 1953, a fire broke out in the factory were Ty was working. A hevy object fell on his hand
did not intend to limit or exempt itself from liability for such death. when he was trying to put out the fire.
DOCTRINE > From Dec. 1953 to Feb. 6, 1954 Ty received treatment at the Nat’l Orthopedic Hospital for six listed
In other words, where the death or injury is not the natural or probable result of the insured's voluntary act, injuries. The attending surgeon certified that these injuries would cause the temporary total disability of
or if something unforeseen occurs in the doing of the act which produces the injury, the resulting death is Ty’s left hand.
within the protection of policies insuring against death or injury from accident. > Insurance companies refused to pay Ty’s claim for compensation under the policies by reason of said
disability of his left hand. Ty filed a complaint in the municipal court who decided in his favor.
Sun v CA G.R. No. 92383 July 17, 1992 > CFI reversed on the ground that under the uniform terms of the policies, partial disability due to loss of
J. Cruz either hand of the insured, to be compensable must be the result of amputation.
Facts: Issue:
Lim accidentally killed himself with his gun after removing the magazine, showing off, pointing the gun at Whether or not Ty should be indemnified under his accident policies.
his secretary, and pointing the gun at his temple. The widow, the beneficiary, sued the petitioner and won Held.
200,000 as indemnity with additional amounts for other damages and attorney’s fees. This was sustained in NO.
the Court of Appeals then sent to the Supreme court by the insurance company. SC already ruled in the case of Ty v. FNSI that were the insurance policies define partial disability as loss
Issue: of either hand by amputation through the bones of the wrist, the insured cannot recover under said policies
1. Was Lim’s widow eligible to receive the benefits? for temporary disability of his left hand caused by the fractures of some fingers. The provision is clear
2. Were the other damages valid? enough to inform the party entering into that contract that the loss to be considered a disability entitled to
Held: indemnity, must be severance or amputation of the affected member of the body of the insured.
1. Yes 2. No
Ratio: 1. There was an accident. Calanoc v. CAG.R. No. L-8151 December 16, 1955J.
De la Cruz v. Capital Insurance says that "there is no accident when a deliberate act is performed unless
some additional, unexpected, independent and unforeseen happening occurs which produces or brings Bautista Angelo
about their injury or death." This was true when he fired the gun. Doctrine: In case of ambiguity in an insurance contract covering accidental death, the Supreme Court held
Under the insurance contract, the company wasn’t liable for bodily injury caused by attempted suicide or that such terms shall be construed strictly against the insurer and liberally in favor of the insured in order to
by one needlessly exposing himself to danger except to save another’s life. effect the purpose of indemnity.
Lim wasn’t thought to needlessly expose himself to danger due to the witness testimony that he took steps Facts: Melencio Basilio, a watchman of the Manila Auto Supply, secured a life insurance policy from the
to ensure that the gun wasn’t loaded. He even assured his secretary that the gun was loaded. Philippine American Insurance Company in the amount of P2,000 to which was attached a supplemental
There is nothing in the policy that relieves the insurer of the responsibility to pay the indemnity agreed contract covering death by accident. He later died from a gunshot wound on the occasion of a robbery
upon if the insured is shown to have contributed to his own accident. committed; subsequently, his widow was paid P2,000 representing the face value of the policy. The widow
2. “In order that a person may be made liable to the payment of moral damages, the law requires that his act demanded the payment of the additional sum of P2,000 representing the value of the supplemental policy
be wrongful. The adverse result of an action does not per se make the act wrongful and subject the act or to which the company refused because the deceased died by murder during the robbery and while making an
the payment of moral damages. The law could not have meant to impose a penalty on the right to litigate; arrest as an officer of the law which were expressly excluded in the contract. The company’s contention
such right is so precious that moral damages may not be charged on those who may exercise it erroneously. which was upheld by the Court of Appeals provides that
For these the law taxes costs.” The circumstances surrounding Basilio’s death was caused by one of the risks excluded by the
If a party wins, he cannot, as a rule, recover attorney's fees and litigation expenses, since it is not the fact of supplementary contract which exempts the company from liability.
winning alone that entitles him to recover such damages of the exceptional circumstances enumerated in
Art. 2208. Otherwise, every time a defendant wins, automatically the plaintiff must pay attorney's fees Issue: Is the Philippine American Life Insurance Co. liable to the petitioner for the amount covered by the
thereby putting a premium on the right to litigate which should not be so. For those expenses, the law supplemental contract?
deems the award of costs as sufficient.”
Page 12 of 39
Held: Yes. While said insurance policy was in full force and effect, the insured Carlie Surposa, died on October 18,
The circumstances of Basilio’s death cannot be taken as purely intentional on the part of Basilio to expose 1988 as a result of a stab wound inflicted by one of the three (3) unidentified men without provocation and
himself to the danger. There is no proof that his death was the result of intentional killing because there is warning on the part of the former as... he and his cousin, Winston Surposa, were waiting for a ride on their
the possibility that the malefactor had fired the shot merely to scare away the way home along Rizal-Locsin Streets, Bacolod City after attending the celebration of the "Maskarra Annual
people around. In this case, the company’s defense points out that Basilio’s is included among the risks Festival."... private respondent and the other beneficiaries of said insurance policy filed a written notice of
excluded in the supplementary contract; however, the terms and phraseology of the exception clause should claim with the petitioner insurance company which denied said claim contending that murder and assault
be clearly expressed within the understanding of the insured. Art. 1377 of the New Civil Code provides that are not within the scope of the coverage of the insurance... policy.
in case ambiguity, uncertainty or obscurity in the interpretation of the terms of the contract, it shall be Insurance Commission ruled in favor of insured/beneficiaries
construed against the party who caused such obscurity. Applying this to the situation, the ambiguous or On February 24, 1989, private respondent filed a complaint with the Insurance Commission
obscure terms in the insurance policy are to be construed strictly against the insurer and liberally in favor of "In the light of the foregoing, we find respondent liable to pay complainant the sum of P15,000.00
the insured party. The reason is to ensure the protection of the insured since these insurance contracts are representing the proceeds of the policy with interest. As no evidence was submitted to prove the claim for
usually arranged and employed by experts and legal advisers acting exclusively in the interest of the mortuary aid in the sum of P1,000.00, the same... cannot be entertained.
insurance company. As long as insurance companies insist upon the use of ambiguous, intricate and On July 11, 1991, the appellate court affirmed said decision.
technical provisions, which conceal their own intentions, the courts must, in fairness to those who purchase petitioner filed this petition alleging grave abuse of discretion on the part of the appellate court in applying
insurance, construe every ambiguity in favor of the insured. the principle of "expresso unius exclusion alterius" in a personal accident insurance policy... since death
resulting from murder and/or assault are impliedly excluded in said insurance policy considering that the
cause of death of the insured was not accidental but rather a deliberate and intentional act of the assailant in
Biagtan vs. The Insular Life Assurance Company, Ltd. (winner) 44 SCRA 58 killing the former as indicated by the location... of the lone stab wound on the insured.
Facts: Therefore, said death was committed with deliberate intent which, by the very nature of a personal accident
Juan S. Biagtan was insured with defendant Insular Life Assurance Company under Policy No. 398075 for insurance policy, cannot be indemnified.
the sum of P5,000.00 and, under a supplementary contract denominated "Accidental Death Benefit Clause, Issues:
for an additional sum of P5,000.00 if "the death of the Insured resulted directly from bodily injury effected WON the death of the insured was committed...... with deliberate intent which, by the very nature of a
solely through external and violent means sustained in an accident . . . and independently of all other personal accident insurance policy, cannot be indemnified
causes." The clause, however, expressly provided that it would not apply where death resulted from an Ruling:
injury "intentionally inflicted by a third party."library We do not agree.
On the night of May 20, 1964 or during the first hours of the following day a band of robbers entered the In the case at bar, it cannot, be pretended that Carlie Surposa died in the course of an assault or murder as a
house of the insured Juan S. Biagtan. result of his voluntary act considering the very nature of these crimes.
Issue: the personal accident insurance policy, involved herein specifically enumerated only ten (10) circumstances
Whether the wounds received by the insured at the hands of the robbers were inflicted intentionally. wherein no liability attaches to petitioner insurance company for any injury, disability or loss suffered by
Held: the insured as a result of any of the... stipulated causes. The principle of "expresso unius exclusio alterius"
Yes. But where a gang of robbers enter a house and coming face to face with the owner, even if -- the mention of one thing implies the exclusion of another thing -- is therefore applicable in the instant
unexpectedly, stab him repeatedly, it is contrary to all reason and logic to say that his injuries are not case since murder and assault, not having been expressly included in the enumeration of the...
intentionally inflicted, regardless of whether they prove fatal or not. As it was, in the present case they did circumstances that would negate liability in said insurance policy cannot be considered by implication to
prove fatal, and the robbers have been accused and convicted of the crime of robbery with homicide. Under discharge the petitioner insurance company from liability for any injury, disability Or loss suffered by the
the circumstance, the insurance company was correct in refusing to pay the additional sum of P2,000.00 insured.
under the accidental death benefit clause which expressly provided that it would not apply where death Thus, the failure of the petitioner insurance company... to include death resulting from murder or assault
resulted from an injury "intentionally" inflicted by a third party. among the prohibited risks leads inevitably to the conclusion that it did not intend to limit or exempt itself
from liability for such death.
FINMAN GENERAL ASSURANCE CORPORATION v. CA, GR No. 100970, 1992-09-02 Principles:
Facts: The terms 'accident' and 'accidental', as used in insurance contracts have not acquired any technical
October 22, 1986, deceased Carlie Surposa was insured with petitioner Finman General Assurance meaning, and are construed by the courts in their ordinary and common acceptation. Thus, the terms have
Corporation under Finman General Teachers Protection Plan Master Policy No. 2005 and Individual Policy been taken to mean that which happen by chance or... fortuitously, without intention and design, and which
No. 08924 with his parents, spouses is unexpected, unusual, and unforeseen. An accident is an event that takes place without one's foresight or
Julia and Carlos Surposa and brothers Christopher, Charles, Chester and Clifton, all surnamed Surposa, as expectation -- an event that proceeds from an unknown cause, or is an unusual effect of a known cause and,
beneficiaries. therefore,... not expected."

Page 13 of 39
The generally accepted rule is that, death or injury does not result from accident or accidental means within Moreover, taking into account the well-known rule that ambiguities or obscurities must strictly be
the terms of an accident-policy if it is the natural result of the insured's voluntary act, unaccompanied by interpreted against the party that cause them, the memorandum of warranty invoked by the insurer bars the
anything unforeseen except the death or... injury. latter from questioning the existence of the appliances called for, since its initial expression “the undernoted
appliances for the extinction of fire being kept on the premises insured hereby..” admits of the interpretation
Qua Chee Gan v. Law Union Rock - Breach of Warranty as an admission of the existence of such appliances which insurer cannot now contradict, should the parole
98 PHIL 85 evidence apply.
Facts:
> Qua Chee Gan, a merchant, owned 4 warehouses in Albay which were used for the storage or copra and (2) Whether or not the insured violated the hemp warranty provision against the storage of gasoline since
hemp in which the appelle deals with exclusively. insured admitted there were 36 cans of gasoline in Bodega 2 which was a separate structure and not
> The warehouses together with the contents were insured with Law Union since 1937 and the loss made affected by the fire.
payable to PNB as mortgagee of the hemp and copra.
> A fire of undetermined cause broke out in July 21, 1940 and lasted for almost 1 whole week. It is well to note that gasoline is not specifically mentioned among the prohibited articles listed in the so-
> Bodegas 1, 3, and 4 including the merchandise stored were destroyed completely. called hemp warranty. The clause relied upon by the insurer speaks of “oils”. Ordinarily, oils mean
> Insured then informed insurer of the unfortunate event and submitted the corresponding fire claims, lubricants and not gasoline or kerosene. Here again, by reason of the exclusive control of the insurance
which were later reduced to P370T. company over the terms of the contract, the ambiguity must be held strictly against the insurer and liberally
> Insurer refused to pay claiming violations of the warranties and conditions, filing of fraudulent claims in favor of the insured, specially to avoid a forfeiture.
and that the fire had been deliberately caused by the insured.
> Insured filed an action before CFI which rendered a decision in favor of the insured. Furthermore, the gasoline kept was only incidental to the insured’s business. It is a well settled rule that
keeping of inflammable oils in the premises though prohibited by the policy does NOT void it if such
Issues and Resolutions: keeping is incidental to the business. Also, the hemp warranty forbade the storage only in the building to
(1) Whether or not the policies should be avoided for the reason that there was a breach of warranty. which the insurance applies, and/or in any building communicating therewith; and it is undisputed that no
gasoline was stored in the burnt bodegas and that Bodega No. 2 which was where the gasoline was found
Under the Memorandum of Warranty, there should be no less than 1 hydrant for each 150 feet of external stood isolated from the other bodegas.
wall measurements of the compound, and since bodegas insured had an external wall per meter of 1640
feet, the insured should have 11 hydrants in the compound. But he only had 2. Del Rosario v. Equitable Insurance - Life Insurance Policy
118 PHIL 349
Even so, the insurer is barred by estoppel to claim violation of the fire hydrants warranty, because knowing Facts:
that the number of hydrants it demanded never existed from the very beginning, appellant nevertheless > Equitable Insurance issued a life Insurance policy to del Rosario binding itself to pay P1,000 to P3,000
issued the policies subject to such warranty and received the corresponding premiums. The insurance as indemnity.
company was aware, even before the policies were issued, that in the premises there were only 2 hydrants > Del Rosario died in a boating accident. The heirs filed a claim and Equitable paid them P1,000.
and 2 others were owned by the Municipality, contrary to the requirements of the warranties in question. > The heir filed a complaint for recovery of the balance of P2,000, claiming that the insurere should pay
him P3,000 as stated in the policy.
It should be close to conniving at fraud upon the insured to allow the insurer to claim now as void the Issue:
policies it issued to the insured, without warning him of the fatal defect, of which the insurer was informed, Whether or not the heir is entitled to recover P3,000.
and after it had misled the insured into believing that the policies were effective. Held:
YES.
Accdg to American Jurisprudence: It is a well-settled rule that the insurer at the time of the issuance of a Generally accepted principles or ruling on insurance, enunciate that where there is an ambiguity with
policy has the knowledge of existing facts, which if insisted on, would invalidate the contract from its very respect to the terms and conditions of the policy, the same shall be resolved against the one responsible
inception, such knowledge constitutes a waiver of conditions in the contract inconsistent with known facts, thereof. The insured has little, if any, participation in the preparation of the policy. The interpretation of
and the insurer is stopped thereafter from asserting the breach of such conditions. The reason for the rule obscure stipulations in a contract should not favor the party who cause the obscurity.
is: To allow a company to accept one’s money for a policy of insurance which it knows to be void and of no
effect, though it knows as it must that the insured believes it to be valid and binding is so contrary to the Insurance Case Digest: Del Rosario v. Equitable Ins. and Casualty Co., Inc. (1963)
dictates of honesty and fair dealing, as so closely related to positive fraud, as to be abhorrent to fair-minded G.R. No. L-16215 June 29, 1963
men. It would be to allow the company to treat the policy as valid long enough to get the premium on it, Lessons Applicable: Ambiguous Provisions Interpreted Against Insurer (Insurance)
and leave it at liberty to repudiate it the next moment. FACTS:

Page 14 of 39
April 13, 1957: Simeon del Rosario, father of the insured who died from drowning filed a claim for Ironically, during the trial, Verendia admitted that it was not Robert Garcia who signed the lease contract
payment with Equitable Ins. and Casualty Co., Inc. but it refused to pay more than P1,000 php so a case but it was Marcelo Garcia cousin of Robert, who had also been paying the rentals all the while. Verendia,
was filed with the RTC for the P2,000 balance stating that under the policy they are entitled to P1,000 to however, failed to explain why Marcelo had to sign his cousin's name when he in fact he was paying for the
P3,000 as indemnity rent and why he (Verendia) himself, the lessor, allowed such a ruse. Fidelity's conclusions on these proven
RTC: entitled to recover P3,000 - policy does not positively state any definite amount, there is an ambiguity facts appear, therefore, to have sufficient bases: Verendia concocted the lease contract to deflect
in this respect in the policy, which ambiguity must be interpreted in favor of the insured and strictly against responsibility for the fire towards an alleged "lessee", inflated the value of the property by the alleged
the insurer so as to allow greater indemnity monthly rental of P6,500) when in fact, the Provincial Assessor of Rizal had assessed the property's fair
ISSUE: W/N Simeon is entitled to recover P3,000 market value to be only P40,300.00, insured the same property with two other insurance companies for a
HELD: YES. total coverage of around P900,000, and created a dead-end for the adjuster by the disappearance of Robert
terms in an insurance policy, which are ambiguous, equivocal or uncertain are to be construed strictly Garcia.
against, the insurer, and liberally in favor of the insured so as to effect the dominant purpose of indemnity Basically a contract of indemnity, an insurance contract is the law between the parties. Its terms and
or payment to the insured, especially where a forfeiture is involved conditions constitute the measure of the insurer's liability and compliance therewith is a condition
reason for this rule is that the "insured usually has no voice in the selection or arrangement of the words precedent to the insured's right to recovery from the. As it is also a contract of adhesion, an insurance
employed and that the language of the contract is selected with great care and deliberation by expert and contract should be liberally construed in favor of the insured and strictly against the insurer company which
legal advisers employed by, and acting exclusively in the interest of, the insurance company usually prepares it.
Considering, however, the foregoing discussion pointing to the fact that Verendia used a false lease contract
Verendia v. CA - Insurance Policy to support his claim under Fire Insurance Policy, the terms of the policy should be strictly construed against
217 SCRA 1993 the insured. Verendia failed to live by the terms of the policy, specifically Section 13 thereof which is
Facts: expressed in terms that are clear and unambiguous, that all benefits under the policy shall be forfeited "if
> Fidelity and Surety Insurance Company (Fidelity) issued Fire Insurance Policy No. F-18876 effective the claim be in any respect fraudulent, or if any false declaration be made or used in support thereof, or if
between June 23, 1980 and June 23, 1981 covering Rafael (Rex) Verendia's residential in the amount of any fraudulent means or devises are used by the Insured or anyone acting in his behalf to obtain any benefit
P385,000.00. Designated as beneficiary was the Monte de Piedad & Savings Bank. under the policy". Verendia, having presented a false declaration to support his claim for benefits in the
> Verendia also insured the same building with two other companies, namely, The Country Bankers form of a fraudulent lease contract, he forfeited all benefits therein by virtue of Section 13 of the policy in
Insurance for P56,000.00 and The Development Insurance for P400,000.00. the absence of proof that Fidelity waived such provision
> While the three fire insurance policies were in force, the insured property was completely destroyed by There is also no reason to conclude that by submitting the subrogation receipt as evidence in court, Fidelity
fire. bound itself to a "mutual agreement" to settle Verendia's claims in consideration of the amount of
> Fidelity appraised the damage amounting to 385,000 when it was accordingly informed of the loss. P142,685.77. While the said receipt appears to have been a filled-up form of Fidelity, no representative of
Despite demands, Fidelity refused payment under its policy, thus prompting Verendia to file a complaint for Fidelity had signed it. It is even incomplete as the blank spaces for a witness and his address are not filled
the recovery of 385,000 up. More significantly, the same receipt states that Verendia had received the aforesaid amount. However,
> Fidelity, averred that the policy was avoided by reason of over-insurance, that Verendia maliciously that Verendia had not received the amount stated therein, is proven by the fact that Verendia himself filed
represented that the building at the time of the fire was leased under a contract executed on June 25, 1980 the complaint for the full amount of P385,000.00 stated in the policy. It might be that there had been efforts
to a certain Roberto Garcia, when actually it was a Marcelo Garcia who was the lessee. to settle Verendia's claims, but surely, the subrogation receipt by itself does not prove that a settlement had
been arrived at and enforced. Thus, to interpret Fidelity's presentation of the subrogation receipt in evidence
Issue: as indicative of its accession to its "terms" is not only wanting in rational basis but would be substituting
Whether or not Verendia can claim on the insurance despite the misrepresentation as to the lessee and the the will of the Court for that of the parties
overinsurance.

Held:
NOPE.
The contract of lease upon which Verendia relies to support his claim for insurance benefits, was entered Fortune Insurance And Surety Co., Inc. V. CA
into between him and one Robert Garcia, a couple of days after the effectivity of the insurance policy.
When the rented residential building was razed to the ground, it appears that Robert Garcia was still within
the premises. However, according to the investigation by the police, the building appeared to have "no Lessons Applicable: Stipulations Cannot Be Segregated (Insurance)
occupants" and that Mr. Roberto Garcia was "renting on the otherside of said compound" These pieces of
evidence belie Verendia's uncorroborated testimony that Marcelo Garcia whom he considered as the real
lessee, was occupying the building when it was burned. FACTS:
Page 15 of 39
 Producers Bank of the Philippines insured with Fortune Insurance and Surety Co. P725,000 which
Misamis Lumber Corp. V. Capital Ins. And Surety Co.,
was lost during a robbery of Producer's armored vehicle while it was in transit from Pasay City City to
its Makati head office.
Lessons Applicable: Judicial Construction Cannot Alter Terms (Insurance)
 The armored car was driven by Benjamin Magalong Y de Vera, escorted by Security Guard
Saturnino Atiga Y Rosete.
FACTS:
 After an investigation conducted by the Pasay police authorities, the driver Magalong and guard
 Misamis Lumber Corporation (Misamis), formerly Lanao Timber Mills, Inc., insured its Ford
Atiga were charged, together with Edelmer Bantigue Y Eulalio, Reynaldo Aquino and John Doe, with
Falcon motor car with Capital Insurance & Surety Company (Capital)
violation of P.D. 532 (Anti-Highway Robbery Law)
 November 25, 1961 11 pm: The car broke when it hit a hollow block lying alongside the water
 Upon claiming, Fortune refused stating that it is not liable since under the general exceptions of
hole which the driver did not see because the on-coming car did not dim its light
the policy:
 The car was towed and repaired by Morosi Motors costing P302.27
 any loss caused by any dishonest, fraudulent or criminal act of the insured or any officer,
 November 29, 1961: After the repairs were made, Misamis made a report to Capital who only
employee, partner, director, trustee or authorized representative of the Insured whether acting alone or
admits liability of P150
in conjunction with others. . . .
 CFI: paragraph 4 of the policy is clear and specific and leaves no room for interpretation that the
 RTC: favored Producers Bank since Driver and Security Guard were merely assigned
repair liability is limited to P150
 CA: Affirmed RTC
ISSUE: W/N Misamis is entitled to an amount exceeding P150
ISSUE: W/N the driver and security guard are employees under the general exception

HELD: NO.
HELD: YES. Petition is granted.
 insurance contract may be rather onerous (one-sided) but that in itself does not justify the
 It is clear to us that insofar as Fortune is concerned, it was its intention to exclude and exempt
abrogation of its express terms, terms which the insured accepted or adhered to and which is the law
from protection and coverage losses arising from dishonest, fraudulent, or criminal acts of persons
between the contracting parties
granted or having unrestricted access to Producers' money or payroll. When it used then the term
Misamis Lumber vs. Capital Insurance, G.R. L-21380, May 20, 1966
"employee," it must have had in mind any person who qualifies as such as generally and universally
understood, or jurisprudentially established in the light of the four standards in the determination of the
Facts:
employer-employee relationship, 21 or as statutorily declared even in a limited sense as in the case of
Article 106 of the Labor Code which considers the employees under a "labor-only" contract as
The vehicle owned by the petitioner was covered by an insurance policy issued by the respondent. In such
employees of the party employing them and not of the party who supplied them to the employer
policy it contained instructions and details on how to proceed with the claim for repairs.
 Producers entrusted the three with the specific duty to safely transfer the money to its head office,
with Alampay to be responsible for its custody in transit; Magalong to drive the armored vehicle which
When the time came for the vehicle to be repaired, the petitioner took it upon himself to have the vehicle
would carry the money; and Atiga to provide the needed security for the money, the vehicle, and his
repaired without the authority from the insurer, in the policy if such an event is to occur, and a repair was
two other companions.
done without the authority of the insurer, its liability is limited only to 150 pesos. The repair bill exceeded
 A "representative" is defined as one who represents or stands in the place of another; one who
such amount.
represents others or another in a special capacity, as an agent, and is interchangeable with "agent."
Page 16 of 39
Hence this petition by White Gold.

Upon filing of claim the insurer refused payment in excess of 150 pesos. Issues:

1. Is Steamship Mutual, a P & I Club, engaged in the insurance business in the Philippines?
Issue(s):
2. Does Pioneer need a license as an insurance agent/broker for Steamship Mutual?

Held: Yes. Petition granted.


1.) Can the respondent be made to reimburse the petitioner for the actual cost of repairs which exceed the
repair limit amount? Ratio:

White Gold insists that Steamship Mutual as a P & I Club is engaged in the insurance business. To buttress
Ruling: its assertion, it cites the definition as “an association composed of shipowners in general who band together
for the specific purpose of providing insurance cover on a mutual basis against liabilities incidental to
shipowning that the members incur in favor of third parties.”
The lower court’s recourse to legal hermeneutics is not called for because paragraph 4 of the policy is clear
They argued that Steamship Mutual’s primary purpose is to solicit and provide protection and indemnity
and specific and leaves no room for interpretation. The interpretation is even unjustified because it opposes coverage and for this purpose, it has engaged the services of Pioneer to act as its agent.
what was specifically stipulated. Thus it will be observed that the policy drew out not only the limits of the
Respondents contended that although Steamship Mutual is a P & I Club, it is not engaged in the insurance
insurer’s liability but also the mechanics that the insured had to follow to be entitled to full indemnity for business in the Philippines. It is merely an association of vessel owners who have come together to provide
repairs. The option to undertake repairs is accorded to the insurance company per paragraph 2. The said mutual protection against liabilities incidental to shipowning.
company was deprived of the option because the insured took it upon itself to have the repairs made, and Is Steamship Mutual engaged in the insurance business?
only notified the insurer when the repairs are done. As a consequence, paragraph 4, which limits the
A P & I Club is “a form of insurance against third party liability, where the third party is anyone other than
company’s liability to P150.00 appliesWEEK 4 the P & I Club and the members.” By definition then, Steamship Mutual as a P & I Club is a mutual
White Gold v Pioneer G.R. No. 154514. July 28, 2005 insurance association engaged in the marine insurance business.
Facts: The records reveal Steamship Mutual is doing business in the country albeit without the requisite certificate
White Gold procured a protection and indemnity coverage for its vessels from The Steamship Mutual of authority mandated by Section 187 of the Insurance Code. It maintains a resident agent in the
through Pioneer Insurance and Surety Corporation. White Gold was issued a Certificate of Entry Philippines to solicit insurance and to collect payments in its behalf. Steamship Mutual even renewed its P
and Acceptance. Pioneer also issued receipts. When White Gold failed to fully pay its accounts, Steamship & I Club cover until it was cancelled due to non-payment of the calls. Thus, to continue doing business
Mutual refused to renew the coverage. here, Steamship Mutual or through its agent Pioneer, must secure a license from the Insurance Commission.

Steamship Mutual thereafter filed a case against White Gold for collection of sum of money to recover the Since a contract of insurance involves public interest, regulation by the State is necessary. Thus, no insurer
unpaid balance. White Gold on the other hand, filed a complaint before the Insurance Commission or insurance company is allowed to engage in the insurance business without a license or a certificate of
claiming that Steamship Mutual and Pioneer violated provisions of the Insurance Code. authority from the Insurance Commission.

The Insurance Commission dismissed the complaint. It said that there was no need for Steamship Mutual 2. Pioneer is the resident agent of Steamship Mutual as evidenced by the certificate of registration issued by
to secure a license because it was not engaged in the insurance business and that it was a P & I club. the Insurance Commission. It has been licensed to do or transact insurance business by virtue of the
Pioneer was not required to obtain another license as insurance agent because Steamship Mutual was not certificate of authority issued by the same agency. However, a Certification from the Commission states
engaged in the insurance business. that Pioneer does not have a separate license to be an agent/broker of Steamship Mutual.

The Court of Appeals affirmed the decision of the Insurance Commissioner. In its decision, Although Pioneer is already licensed as an insurance company, it needs a separate license to act as
the appellate court distinguished between P & I Clubs vis-à-vis conventional insurance. The appellate court insurance agent for Steamship Mutual. Section 299 of the Insurance Code clearly states:
also held that Pioneer merely acted as a collection agent of Steamship Mutual.
Page 17 of 39
SEC. 299 No person shall act as an insurance agent or as an insurance broker in the solicitation or which such insurer may be liable under a contract of suretyship, or for which a reinsurer may be used under
procurement of applications for insurance, or receive for services in obtaining insurance, any commission any contract or reinsurance it may have entered into, or for which a mutual benefit association may be held
or other compensation from any insurance company doing business in the Philippines or any agent thereof, liable under the membership certificates it has issued to its members, where the amount of any such loss,
without first procuring a license so to act from the Commissioner… damage or liability, excluding interest, costs and attorney's fees, being claimed or sued upon any kind of
insurance, bond, reinsurance contract, or membership certificate does not exceed in any single claim
Philam v Arnaldo G.R. No. 76452 July 26, 1994 one hundred thousand pesos.
Facts: This was, however, regarding complaints filed by the insured against the Insurance company.
One Ramon Paterno complained about the unfair practices committed by the company against its agents, Also, the insurance code only discusses the licensing requirements for agents and brokers. The Insurance
employees and consumers. The Commissioner called for a hearing where Paterno was required to specify Code does not have provisions governing the relations between insurance companies and their agents.
which acts were illegal. Paterno then specified that the fees and charges stated in the Contract of Agency
between Philam and its agents be declared void. Philam, on the other hand, averred that there Paterno must Investment Planning Corporation of the Philippines v. Social Security Commission- “that an insurance
submit a verified formal complaint and that his letter didn’t contain information Philam was seeking from company may have two classes of agents who sell its insurance policies: (1) salaried employees who keep
him. Philam then questioned the Insurance Commission’s jurisdiction over the matter and submitted a definite hours and work under the control and supervision of the company; and (2) registered
motion to quash. The commissioner denied this. Hence this petition. representatives, who work on commission basis.”
Issue: Whether or not the resolution of the legality of the Contract of Agency falls within the jurisdiction of The agents under the 2nd sentence are governed by the Civil Code laws on agency. This means that the
the Insurance Commissioner. regular courts have jurisdiction over this category.
Held: No. Petition granted.

Ratio: FILIPINAS COMPANIA DE SEGUROS vs. CHRISTERN HUENEFELD and CO., INC. 89 Phil 54
According to the Insurance code, the Insurance Commissioner was authorized to suspend, directors,
officers, and agents of insurance companies. In general, he was tasked to regulate the insurance business, FACTS:
which includes:

(2) The term "doing an insurance business" or "transacting an insurance business," within the On October 1, 1941, the respondent corporation, Christern Huenefeld and Co., Inc., after payment of
meaning of this Code, shall include corresponding premium, obtained from the petitioner, Filipinas Cia de Seguros fire policy covering
merchandise contained in a building located at Binondo, Manila. On February 27, 1942 or during the
(a) making or proposing to make, as insurer, any insurance contract; Japanese military occupation, the building and insured merchandise were burned. In due time, the
respondent submitted to the petitioner its claim under the policy. The petitioner refused to pay the claim on
(b) making, or proposing to make, as surety, any contract of suretyship as a vocation and not as the ground that the policy in favor of the respondent that ceased to be a force on the date the United States
merely incidental to any other legitimate business or activity of the surety; (c) doing any kind of business, declared war against Germany, the respondent corporation (through organized under and by virtue of the
including a reinsurance business, specifically recognized as constituting the doing of an insurance business laws of Philippines) being controlled by German subjects and the petitioner being a company under
within the meaning of this Code; (d) doing or proposing to do any business in substance equivalent to any American jurisdiction when said policy was issued on October 1, 1941. The theory of the petitioner is that
of the foregoing in a manner designed to evade the provisions of this Code. (Insurance Code, Sec. 2[2]) the insured merchandise was burned after the policy issued in 1941 had ceased to be effective because the
outbreak of the war between United States and Germany on December 10, 1941, and that the payment
The contract of agency between Philamlife and its agents wasn’t included with the Commissoner’s power made by the petitioner to the respondent corporation during the Japanese military occupation was under
to regulate the business. Hence, the Insurance commissioner wasn’t vested with jurisidiction under the rule pressure.
“expresio unius est exclusionalterius”. ISSUE:
The respondent contended that the commissioner had the quasi-judicial power to adjudicate under Section
416 of the Code. It stated: Whether or not the respondent corporation is a corporation of public enemy.
RULING:
The Commissioner shall have the power to adjudicate claims and complaints involving any loss, damage or
liability for which an insurer may be answerable under any kind of policy or contract of insurance, or for
Page 18 of 39
Since the majority of stockholders of the respondent corporation were German subjects, the respondent ISSUE: W/N Gonzaga's widow can claim despite the absence of premium payment during the outbreak of
became an enemy of the state upon the outbreak of the war between US and Germany. The English and
American cases relied upon by the Court of Appeals lost in force upon the latest decision of the Supreme the war
Court of US in which the control test has adopted.
Since World War I, the determination of enemy nationality of corporations has been discussed in many
countries, belligerent and neutral. A corporation was subject to enemy legislation when it was controlled by HELD: NO. Affirmed
enemies, namely managed under the influence of individuals or corporations themselves considered as
enemies…  Non-payment at the day involves absolute forfeiture is such be the terms of the contract
The Philippine Insurance Law (Act No 2427, as amended), in Section 8, provides that “anyone except a
public enemy may be insured”. It stands to reason that an insurance policy ceases to be allowable as soon  failure to notify the postal address during the war is not an excuse
as an insured becomes a public enemy.  There is no duty when the law forbids and there is no obligation without corresponding
The respondent having an enemy corporation on December 10, 1941, the insurance policy issued in its
right enjoyed by another
favor on October 1, 1941, by the petitioner had ceased to be valid and enforceable, and since the insured
good were burned during the war, the respondent was not entitled to any indemnity under said policy from  opening of an interim office partook of the nature of the privilege to the policy holders to keep
the petitioner. However, elementary rule of justice (in the absence of specific provisions in the Insurance their policies operative rather than a duty to them under the contract
Law) require that the premium paid by the respondent for the period covered by its policy from December
11, 1941, should be returned by the petitioner.
WEEK 5

Sales De Gonzaga V. Crown Life Insurance Co. Palileo V. Cosio (1955)

FACTS: FACTS:

 September 26, 1939: Crown Life Insurance Co. whose home office is based in Toronto, Canada  Cherie Palileo (debtor-mortgagor) filed a complaint against Beatriz Cosio (creditor-mortgagee)
issued to Ramon Gonzaga through its branch office in Manila a 20-year endowment policy for P15,000 praying that their transaction be one of a loan with an equitable mortgage to secure the payment of the
which had an annual premium of P591. loan. The original counsel of Cosio Atty. Guerrero being appointed Undersecretary of Foreign Affairs
 Payment was only until September 6, 1941 because of the outbreak of the war since Crown is an so she forgot the date of the trial and she was substituted.
enemy corp. order to be closed during the Japanese occupation. However, despite that it offered  it is a loan of P12,000 secured by a "Conditional Sale of Residential Building" with right to
a privilege to accept premium payments in the place of its employee in Ermita but of which Gonzaga repurchase. After the execution of the contract, Cosio insured in her name the building
did not avail. with Associated Insurance & Surety Co. against fire.
 Through the automatic premium loan clause, it continued until June 12, 1943  The building was partly destroyed by fire so she claimed an indemnity of P13,107
 May 1, 1945: It reopened but still Gonzaga did not pay although there was a reinstatement clause  Palileo demanded that the amount of insurance proceeds be credited to her loan
providing certain conditions within three years from the date of lapse on application of the insured  RTC: it is a loan with equitable mortgage so the insurance proceeds should be credited to the loan
 June 27, 1945: Gonzaga died from an accident and refund the overpayment.
 Crown refused to pay because of the lapse of premium payment ISSUE: W/N Cosio as mortgagee is entitled to the insurance proceeds for her own benefit
 RTC: against Gonzaga

Page 19 of 39
HELD: YES. Modify. collection of insurance proceeds shall not be deemed to have compensated the  San Miguel insured the property only as mortgagee.
obligation of the Palileo to Cosio, but bars the Cosio from claiming its payment from the Palileo; and Cosio  Dunn sold the propert to Henry Harding. The insurance was not assigned by Dunn to Harding.
shall pay to Palileo P810 representing the overpayment made by Palileo by way of interest on the loan.  When it was destroyed by fire, the two companies settled with San Miguelto the extent of the
 When the the mortgagee may insure his interest in the property independently of the mortgagor , mortgage credit.
upon the destruction of the property the insurance money paid to the mortgagee will not inure to the  RTC: Absolved the 2 companies from the difference. Henry Harding is not entitled to the
benefit of the mortgagor, and the amount due under the mortgage debt remains unchanged. The difference between the mortgage credit and the face value of the policies.
mortgagee, however, is not allowed to retain his claim against the mortgagor, but it passes by  Henry Harding appealed.
subrogation to the insurer, to the extent of the insurance money paid ISSUE:
 It is true that there are authorities which hold that "If a mortgagee procures insurance on his 1. W/N San Miguel has insurable interest as mortgagor only to the extent of the mortgage credit - YES
separate interest at his own expense and for his own benefit, without any agreement with the 2. W/N Harding has insurable interest as owner - NO
mortgagor with respect thereto, the mortgagor has no interest in the policy, and is not entitled to have
the insurance proceeds applied in reduction of the mortgage debt" But these authorities merely
represent the minority view HELD: affirmed
 section 19 of the Insurance Act:
San Miguel Brewery V. Law Union And Rock Insurance
 a change of interest in any part of a thing insured unaccompanied by a corresponding
change of interest in the insurance, suspends the insurance to an equivalent extent, until the interest in
the thing and the interest in the insurance are vested in the same person
Lessons Applicable:
 section 55:
 Mortgagor (Insurance)
 the mere transfer of a thing insured does not transfer the policy, but suspends it until the
 Measure of Insurable Interest (Insurance)
same person becomes the owner of both the policy and the thing insured
 Effect of Change of Interest in Thing Insured (Insurance)
 Undoubtedly these policies of insurance might have been so framed as to have been "payable to
 Effect of transfer of thing insured (Insurance)
the San Miguel Brewery, mortgagee, as its interest may appear, remainder to whomsoever, during the
Laws Applicable: sec. 16,sec. 19 (now sec. 20),sec. 50,sec.55 (now sec. 58) of the Insurance Code (all old
continuance of the risk, may become the owner of the interest insured." (Sec 54, Act No. 2427.) Such a
law)
clause would have proved an intention to insure the entire interest in the property, not merely the
insurable interest of the San Miguel Brewery, and would have shown exactly to whom the money, in
FACTS:
case of loss, should be paid. But the policies are not so written.
 In the contract of mortgage, the owner P.D. Dunn had agreed, at his own expense, to insure the
 The blame for the situation thus created rests, however, with the Brewery rather than with the
mortgaged property for its full value and to indorse the policies in such manner as to authorize the
insurance companies, and there is nothing in the record to indicate that the insurance companies were
Brewery Company to receive the proceeds in case of loss and to retain such part thereof as might be
requested to write insurance upon the insurable interest of the owner or intended to make themselves
necessary to satisfy the remainder then due upon the mortgage debt. Instead, however, of effecting the
liable to that extent
insurance himself Dunn authorized and requested the Brewery Company to procure insurance on the
 If by inadvertence, accident, or mistake the terms of the contract were not fully set forth in the
property in the amount of P15,000 at Dunn's expense.
policy, the parties are entitled to have it reformed. But to justify the reformation of a contract, the
Page 20 of 39
proof must be of the most satisfactory character, and it must clearly appear that the contract failed to The action by the insurance company of taking the premiums of the insured notwithstanding knowledge of
express the real agreement between the parties violations of the provisions of the policies amounted to waiver of the right to annul the contract of
 In the case now before us the proof is entirely insufficient to authorize reformation. insurance.

Geagonia vs. Court of Appeals, 241 SCRA 152


Gonzalez Lao v. Yek Tong Lin Fire & Marine Insurance - Insurance Premiums
Facts: Geagonia, owner of a store, obtained from Country Bankers 1year fire insurance covering the
stock trading of dry goods. The policy noted the requirement that"
Facts:
3. The insured shall give notice to the Company of any insurance or insurances already effected, or which
may subsequently be effected, covering any of the property or properties consisting of stocks in trade,
> Gonzales was issued 2 fire insurance policies by Yek for 100T covering his leaf tobacco prducts. goods in process and/or inventories only hereby insured, xxx

> They were stored in Gonzales’ building on Soler St., which on Jan. 11, 1928, burned down. The petitioners’ stocks were destroyed by fire. He then filed a claim which was subsequently denied
because the petitioner’s stocks were covered by two other
> Art. 3 of the Insurance policies provided that: “Any insurance in force upon all or part of the things fire insurance policies issued by PFIC. The basis of the private respondent's denial was the petitioner's
alleged violation of Condition 3 of the policy. The Insurance Commission found that the petitioner did not
unsured must be declared in writing by the insured and he (insured) should cause the company to insert or violate Condition 3 as he had no knowledge of the existence of the two fire insurance policies obtained
mention it in the policy. Without such requisite, such policy will be regarded as null and void and the from the PFIC; that it was Cebu Tesing Textiles which procured the PFIC policies w/o informing him or
securing his consent; and that Cebu Tesing Textile, as his creditor, had insurable interest on the stocks.
insured will be deprived of all rights of indemnity in case of loss.”
Issue: Whether or not Geagonia is prohibited from recovering from the Country Bankers ?
> Notwithstanding said provision, Gonzales entered into other insurance contracts. When he sought to
Held: A policy may declare that a violation of specified provisions thereof shall avoid it,otherwise, the
claim from Yek after the fire, the latter denied any liability on the ground of violation of Art. 3 of the said
breach of an immaterial provision does not avoid the policy. To constitute a violation of the “other
policies. insurance” clause, the other insurance must be upon the same subject matter, the same interest therein, and
the same risk
> Gonzales however proved that the insurer knew of the other insurance policies obtained by him long
efore the fire, and the insurer did NOT rescind the insurance polices in question but demanded and
collected from the insured the premiums. Saura Import & Export Co., Inc. V. Philippine International Surety Co., Inc. (1963)

Lessons Applicable: Mortgagor (Insurance)


Issue:
Laws Applicable:
Whether or not Yek is still entitled to annul the contract.
FACTS:
Held:
 Saura Import & Export Co Inc., mortgaged to the Phil. National Bank, a parcel of land.
NO.  The mortgage was amended to guarantee an increased amount, bringing the total mortgaged debt
to P37,000

Page 21 of 39
Under the terms, Eulogio was to pay premiums on a quarterly basin in the amount of P8,062 with a grace
 On the land mortgage is a building owned by Saura Import & Export Co Inc. which was insured
period of 31 days for the payment of each premium subsequent to the first. If any premium was not paid on
with Philippine International Surety (Insurer) even before the mortgage contract so it was required to or before the due date, the policy would be in default, and if the premium remained unpaid until the end of
endorse to mortgagee PNB the grace period, the policy would automatically lapse and become void.
 October 15, 1954: Barely 13 days after the issuance of the fire insurance policy, the insurer Eulogio paid the premiums, however he failed to pay the premium due on January 24, 1998, even after the
cancelled it. Notice of the cancellation was given to PNB (mortgagee). But Saura (insured) was not lapse of the grace period of 31 days. Therefore, lapsed and become void. Eulogio submitted to the
Cabanatuan District Office of Insular Life an application for reinstatement together with the payment of the
informed. premium due on January 24. Insular Life notified Eulogio that his application for reinstatement could not
 April 6, 1955: The building and all its contents worth P40,685.69 were burned so Saura filed a be fully processed because of the unpaid interest thereon. Eulogio was likewise advised by Malaluan
(insurance agent) to pay the premiums that subsequently became due April 1998 and July 1998, plus
claim with the Insurer and mortgagee Bank
interest.
 RTC: dismissed
ISSUE: W/N Philippine International Surety should be held liable for the claim because notice to only the September 17, 1998. Eulogio went to Malaluan's house and paid for the interest which was received by
Malaluan's husband. Later that day, Eulogio died. Without the knowledge of Eulogio's death, Malaluan
mortgagee is not substantial forwarded to the Insular Life the application for reinstatement and the payment made by Eulogio. However,
Insular Life did not act upon such reinstatement for they knew already of Eulogio's death.
HELD:YES. Appealed from is hereby reversed. Philippine International Surety Co., Inc., to pay Saura September 28, 1998, Violeta filed for the insurance claim. Insular Life then informed Violeta in a letter that
Import & Export Co., Inc., P29,000 her claim could not be processed because the insurance policy had lapsed already and that Eulogio failed to
reinstate the same and the payment made done thru Malaluan's husband was, under the insurance policy,
 It was the primary duty of Philippine International Surety to notify the insured, but it did not
was considered a deposit only until approval of the said application. Enclosed to this letter was a check
 If a mortgage or lien exists against the property insured, and the policy contains a clause stating representing the full refund of the past payments made by Eulogio, amounting to P25,417.
that loss, if any, shall be payable to such mortgagee or the holder of such lien as interest may appear,
Violeta requested for a reconsideration of her claim and returned the check to Insular Life. Insular Life
notice of cancellation to the mortgagee or lienholder alone is ineffective as a cancellation of the policy agreed to conduct a re-evaluation of Violeta's claim. Without waiting for the result of the re-evaluation,
to the owner of the property. Violeta filed with the RTC a complaint for death claim benefit alleging the Insular Life was engaged in
unfair claim settlement practice and deliberately failed to act with reasonable promptness on her insurance
 liability attached principally the insurance company, for its failure to give notice of the claim. Violeta claims for the P1.5M insurance, plus interest, attorney's fees and cost of suit.
cancellation of the policy to Saura
Insular Life filed with the RTC an answer with counterclaim saying that the insurance claim was rendered
 it is unnecessary to discuss the errors assigned against appellee bank
void due to non-payment of the premium and countered that Violeta should be ordered to pay attorney's
fees and expenses of litigation incurred by Insular Life.
PNB V CA
RTC declared that Violeta failed to establish by preponderance of evidence her cause of action against the
Violeta Lalican defendant. Violeta failed to establish that the receipt of payment by Malaluan amounted to the reinstatement
vs The Insular Life Insurance Company of the insurance policy. Violeta filed for motion for reconsideration but was denied as well; hence she
Ponente: Chico-Nazario elevated her case for review on Certiorari.

Facts: Issues: (a) Whether the decision of the court can still be reviewed despite having allegedly attained finality
Violeta is the widow of the Eulogio Lalican. During his lifetime, Eulogio applied for an insurance policy and despite the mode of appeal of Violeta erroneous. (b) Whether the RTC has decided the case on a
with Insular Life on April 24, 1997 which contained a 20-year endowment variable income package flexi question of law not in accord with law and applicable decisions of the Supreme Court.
plan worth P500k with two riders worth P500k each. Violeta was named the primary beneficiary.
Ruling:
Petition lacks merit.

Page 22 of 39
RTC's decision has long acquired finality for Violeta failed to file a notice of appeal more than five months (a) The proceeds of life insurance policies paid tobeneficiaries upon the death of the insured ... ." The
after the decision was rendered. Chapter on Corporations does notprovide as above. It is certain that the proceeds of life insurance policies
are exempt. It isnot so certain that the proceeds of life insurance policies paid to corporate
As to the substantial claim of whether there is insurable interest, the Court says that the matter of insurable beneficiaries upon the death of the insured are likewise exempt.T h e s i t u a t i o n w i l l b e b e t t e r
interest is entirely irrelevant and the real point of contention herein is whether Eulogio was able to reinstate elucidated by a brief reference to laws on the samesubject in the
the lapsed insurance policy on his life before his death. United States. The Income Ta x L a w
of 1916 extended to the PhilippineL e g i s l a t u r e , w h e n i t c a m e t o e n a c t
A c t N o . 2 8 3 3 , t o c o p y t h e A m e r i c a n s t a t u t e . Subsequently, the
The Court rules in the negative, for the insurance policy is clear on the procedure of the reinstatement of Congress of the United States enacted its Income Tax Law of 1919, in which certain doubtful
the insurance contract, of which Eulogio has failed to accomplish before his death. As provided by the subjects were clarified. Thus, as to the point before us, it was madeclear, when not only in the part of the
policy, insurance shall be deemed reinstated upon the approval of the insurance policy of the application for law concerning individuals were exemptions providedfor beneficiaries, but also in the part concerning
reinstatement. The approval should be made during the lifetime of the insured, in the case at bar, it wasn’t. corporations, specific reference was
madet o t h e e x e m p t i o n s i n f a v o r o f i n d i v i d u a l s , t h e r e b y m a k i n g t h e s a m
El Oriente Fabrica de Tabacos, Inc. vs. Juan Posadas, Collector of Internal Revenue,G.R. No. 34774, e a p p l i c a b l e t o corporations.
September 21, 1931 T h i s w a s a u t h o r i t a t i v e l y p o i n t e d o u t a n d d e c i d e d b y t h e U n i t e d S t a t e s Supreme
Topic: Court in the case of United States
Insurable Interest in life and health (Section 10) vs
Facts: . Supplee-Biddle Hardware Co. ( [1924], 265U.S., 189), which involved facts quite similar to those before
us.To quote the exact words in the cited case of Chief Justice Taft delivering the opinionof the court: It
Insurer: Manufacturers Life Insurance Co., of Toronto, Canada, thru its is earnestly pressed upon us that proceeds of life insurance paid on the death of the insured
local agent E.E.ElserInsured: A. Velhagen (manager of El Oriente)Beneficiary: El Oriente Fabrica are in fact capital, and cannot be taxed as income … that proceeds of al i f e i n s u r a n c e p o l i c y p a i d
de Tabacos, Inc.El Oriente, in order to protect itself against the loss that it might suffer by reason of the on the death of the insured are not usually
death of its manager, whose death would be a serious loss to El Oriente procured fromthe Insurer an c l a s s e d a s income.C o n s i d e r i n g , t h e r e f o r e , t h e p u r p o r t o f t h e s t i p u l a t e d f a c
insurance policy on the life of the said manager for the sum of 50,000 USD w i t h E l O r i e n t e a s t s , c o n s i d e r i n g t h e uncertainty of Philippine law, and considering the lack of express legislative
t h e d e s i g n a t e d s o l e b e n e f i c i a r y . T h e i n s u r e d h a s n o i n t e r e s t o r participati intention to taxthe proceeds of life insurance policies paid to corporate beneficiaries, particularly when
on in the proceeds of said life insurance policy.El Oriente charged as expenses of its business all the said inthe exemption in favor of individual beneficiaries in the chapter on this subject, the clause
premiums and deducted the same from its gross incomes as reported in its isi n s e r t e d " e x e m p t f r o m t h e
a n n u a l i n c o m e t a x r e t u r n s , w h i c h deductions were allowed by Posadas (Collector of p r o v i s i o n s o f t h i s l a w, " w e d e e m i t r e a s o n a b l e t o h o l d t h e p r o c e e d s o f t h e l i f e i
Internal Revenue) upon showing by El Oriente that such premiums were legitimate expenses of the n s u r a n c e p o l i c y i n q u e s t i o n a s r e p r e s e n t i n g a n i n d e m n i t y a n d n o t taxable
business.U p o n t h e d e a t h o f t h e m a n a g e r , E l O r i e n t e r e c e i v e d a l l t h e p r o c e e d s o f income.T h e f o r e g o i n g p r o n o u n c e m e n t w i l l r e s u l t i n t h e j u d g m e n t b e i n g r e v e r s e d
t h e l i f e insurance policy together with the interest and the dividends accruing thereon, a n d i n another judgment being rendered in favor of El Oriente.
aggregatingP104,957.88. Posadas assessed and levied the sum of P3,148.74 as income tax on
theproceeds of the insurance policy, which was paid by El Oriente under protest. El
Orienteclaiming exemption under Section 4 of the Income Tax Law. El Oriente v. Posadas - Taxability of Insurance Proceeds
56 PHIL 147 (1931)

Facts:
Issue: > El Oriente in order to protect itself against the loss that it might suffer by reason of the death of its
Whether or not the proceeds of insurance taken by a corporation on the life of animportant manager, A. Velhagen, who had had more than thirty-five (35) years of experience in the manufacture of
official to indemnify it against loss in case of his death, are taxable as income under the cigars in the Philippines, procured from the Manufacturers Life Insurance Co., of Toronto, Canada, thru its
Philippine Income Tax Law? local agent E. E. Elser, an insurance policy on the life of the said A. Velhagen for the sum of $50,000,
Ruling: United States currency designating itself as the beneficiary.
The Income Tax Law for the Philippines is Act No. 2833, as amended. In chapter IOn Individuals, is to be > El Oriente paid for the premiums due thereon and charged as expenses of its business all the said
found section 4 premiums and deducted the same from its gross incomes as reported in its annual income tax returns, which
which provides that, "The following incomes shallbe exempt from the provisions of this law: deductions were allowed upon a showing that such premiums were legitimate expenses of its business.

Page 23 of 39
> Upon the death of A. Velhagen in 1929, the El Oriente received all the proceeds of the said life insurance > She admitted to being only the common law wife of the insured.
policy, together with the interests and the dividends accruing thereon, aggregating P104,957.88 > Pascuala, the legal wife, also filed a claim asserting her right as the legal wife. The company then filed
> CIR assessed El Oriente for deficiency taxes because El Oriente did not include as income the proceeds an action for interpleader.
received from the insurance.
Issue:
Issue:
Whether or not the proceeds of insurance taken by a corporation on the life of an important official to Whether or not the common law wife named as beneficiary can collect the proceeds.
indemnify it against loss in case of his death, are taxable as income under the Philippine Income Tax Law
Held:
Held: NO.
NOT TAXABLE. The civil code prohibitions on donations made between persons guilty of adulterous concubinage applies to
In Chapter I of the Tax Code, is to be found section 4 which provides that, "The following incomes shall be insurance contracts. On matters not specifically provided for by the Insurance Law, the general rules on
exempt from the provisions of this law: (a) The proceeds of life insurance policies paid to beneficiaries Civil law shall apply. A life insurance policy is no different from a civil donation as far as the beneficiary is
upon the death of the insured . . ." Section 10, as amended, in Chapter II On Corporations, provides that, concerned, since both are founded on liberality.
"There shall be levied, assessed, collected, and paid annually upon the total net income received in the
preceding calendar year from all sources by every corporation . . .a tax of three per centum upon such DOCTRINE:General rules of civil law should be applied to resolve any void in the Insurance Law,
income . . ." Section 11 in the same chapter, provides the exemptions under the law, but neither here nor in aspronounced in Article 2011 of the NCC.A contract of insurance is personal in
any other section is reference made to the provisions of section 4 in Chapter I. character.FACTS:Buenaventura Cristor Ebrado was married to Pascuala Ebrado. During his lifetime, he was
livingwith his common-law wife, Carponia Ebrado, although he was not legally separated from his
Under the view we take of the case, it is sufficient for our purposes to direct attention to the anomalous and legalwife. Buenaventura was issued by The Insular Life Assurance Co., Ltd., Policy No. 009929 on a
vague condition of the law. It is certain that the proceeds of life insurance policies paid to individual whole-life plan for PhP 5,8882.00 with a rider for Accidental Death Benefits for the same
beneficiaries upon the death of the insured are exempt. It is not so certain that the proceeds of life insurance amount.Buenaventura designated Carponia Ebrado as the revocable beneficiary in his policy. Buenaventura
policies paid to corporate beneficiaries upon the death of the insured are likewise exempt. But at least, it died as a result of an accident when he was hit by a falling branch of a tree. As the insurance policy was
may be said that the law is indefinite in phraseology and does not permit us unequivocally to hold that the still in force, The Insular Life Assurance Co., Ltd stands liable to pay the coverage. Carponia Ebrado, his
proceeds of life insurance policies received by corporations constitute income which is taxable common-law wife, filed with the insurer a claim for the proceeds of the policy as the designated beneficiary
therein. Pascuala Vda. de Ebrado also filed her claim as the widowof the deceased insured. She asserts that
It will be recalled that El Oriente, took out the insurance on the life of its manager, who had had more than she is the one entitled to the insurance proceeds, not the common-law wife. In doubt as to whom the
thirty-five years' experience in the manufacture of cigars in the Philippines, to protect itself against the loss insurance proceeds shall be paid, the insurer, The Insular Life AssuranceCo., Ltd, commenced an action for
it might suffer by reason of the death of its manager. We do not believe that this fact signifies that when the Interpleader before the CFI of Rizal.
plaintiff received P104,957.88 from the insurance on the life of its manager, it thereby realized a net profit
in this amount. It is true that the Income Tax Law, in exempting individual beneficiaries, speaks of the ISSUE: Can a common-law wife of a man who was not legally separated from his legal wife be a
proceeds of life insurance policies as income, but this is a very slight indication of legislative intention. In beneficiary of his life insurance plan?
reality, what the plaintiff received was in the nature of an indemnity for the loss which it actually suffered
because of the death of its manager. HELD/RULING
No.The Insurance Act (RA 2327, as amended) or even the new Insurance Code (PD No. 612, asamended)
does not contain any specific provision grossly resolutory to the question at hand.Section 50 of the
Insurance Act, which provides that “(t)he insurance shall be applied exclusively tothe proper interest of the
person in whose name it is made,” cannot be interpreted that it includesthe beneficiary because a contract of
insurance is personal in character. The general rules of civillaw should be applied to resolve this void in the
Insurance Law.Article 2011 of the New Civil Code states: “The contract of insurance is governed by special
Insular Life vs. Ebrado laws.Matters not expressly provided for in such special laws shall be regulated by this Code.”Article 2012
80 SCRA 181 of the same Code states that, “any person who is forbidden from receiving any donation under Article 739
Facts: cannot be named beneficiary of a life insurance policy by the person who cannot make a donation to him.”
> Buenaventura Ebrado was issued al life plan by Insular Company. He designated Capriona as his Therefore, common-law spouses are barred from receivingdonations from each other.Article 739
beneficiary, referring to her as his wife. provides:“The following donations shall be void:Those made between persons who were guilty of adultery
> The insured then died and Carponia tried to claim the proceeds of the said plan. or concubinage at the time of thedonation;Those made between persons found guilty of the same criminal
Page 24 of 39
offense, in consideration thereof;Those made to a public officer or his wife, descendants or ascendants by because appellants were designated beneficiaries in Consuegra’s life insurance they automatically became
reason of his office. the beneficiaries
also of his retirement insurance.
In the case referred to in No. 1, the action for declaration of nullity may be brought by thespouse of the The provisions clearly indicate that there is need for the employee to file an applicationfor retirement
donor or donee; and the guilt of the donee may be proved by preponderance ofevidence in the same insurance benefits when he becomes a member of the GSIS, and he should state in his application
action.”In essence, a life insurance policy is no different from a civil donation insofar as the beneficiary thebeneficiary of his retirement insurance. Hence, the beneficiary named in the life insurance does not
isconcerned. Both are founded upon the same consideration: liberality. As a consequence, theproscription in automaticallybecome the beneficiary in the retirement insurance unless the same beneficiary in the life
Art. 739 of the New Civil Code should equally operate in life insurance contracts.In the case at bar, the insurance is so designatedin the application for retirement insurance.
requisite proof of common-law relationship between the insured and thebeneficiary has been supplied by The GSIS offers two separate and distinct systems of benefits to its members
the stipulations between the parties in the pre-trial conference. Itwas agreed an stipulated that the deceased paid out from two distinct and separate funds:
insured Buenaventura Ebrado was married toPascuala Ebrado and that, during the lifetime of the deceased (1) Life insurance- proceeds are paid to whoever is named the beneficiary in the life insurance policy as
insured, he was living with hiscommon-law wife, Carponia Ebrado. Based on the foregoing, Carponia provided forin the Insurance Act (Act 2427, as amended), the beneficiary in a life insurance under the GSIS may not
Ebrado is hereby declareddisqualified to be the beneficiary of the late Buenaventura Ebrado in his life necessarily be an heir of the insured. The insured in a life insurance may designate any person as beneficiary
insurance policy. Theproceeds of the policy are hereby held payable to the estate of the deceased insured. unless disqualified to be so under the provisions of the Civil Code. And in the absence of any beneficiary named in the life
insurance policy, the proceeds of the insurance will go to the estate of the insured.
(2) Retirement insurance- primarily intended for the benefit of the employee to provide for his old age, or
Vda. de Consuegra vs. GSIS, DPWH, Rosario Diaz 37 SCRA 315 [1971]MAIN POINT: incapacity, after rendering service in the government for a required number of years. If the employee reaches the age of
FACTS: retirement, he gets the retirement benefits even to the exclusion of the beneficiary or beneficiaries named in his application for
The late Jose Consuegra, at the time of his death, was employed as a shop foreman of DPWH in the province ofSurigao del Norte. retirement insurance. The beneficiary of the retirement insurance can only claim the proceeds of the retirement insurance if the
In his lifetime, he contracted 2 marriages, the first with respondent Rosario Diaz, solemnized in the parish church of San Nicolas de employee dies before retirement.
Tolentino, Surigao, on July 15, 1937 bearing 2 children, Jose Consuegra, Jr. and Pedro Consuegra, but both predeceased their If the employee failed or overlooked to state the beneficiary of his retirement insurance, the retirement
father; and the second with herein petitioner Basilia Berdin, which was contracted in good faith while the first marriage was benefits will accrue to his estate and will be given to his legal heirs in accordance with law, as in the case of
subsisting, on May 1, 1957 in the same parish and municipality, out of which marriage a life insurance if no beneficiary is named in the insurance policy.
were born seven children, namely, Juliana, Pacita, Maria Lourdes, Jose, Rodrigo, Lenida and Luz *(Luis) all
surnamedConsuegra.When Consuegra died the proceeds of his life insurance were paid by the GSIS to petitioner Basilia Berdin B.BOTH.
and her children who were the beneficiaries named in the policy. Having been in the service of the government for 22.5028 years, The proceeds of the retirement insurance of the late Jose Consuegra should be divided equally between Rosario Diaz and his
Consuegra was entitled to retirement insurance benefits in the sum of P6, 304.47, however, he did not designate any beneficiary second wife Basilia Berdin and his children by her. The fact that the second marriage was contracted in good faith, the only
who would receive the retirement insurance benefits due to him. Respondent Diaz, the widow (1st), filed a claim with the GSIS just and equitable solution in this case would be to recognize the right of the second wife to her share of
asking that the retirement insurance benefits be paid to her as the only legal heir. Petitioner Berdin and her children, likewise, filed a one-half in the property acquired by her and her husband, and consider the other half as pertaining to the
similar claim with the GSIS, asserting that being the beneficiaries named in the life insurance policy,they are the only ones conjugal partnership of the first marriage.
entitled The decision appealed from is affirmed
to receive the retirement insurance benefits due the deceased. GSIS: Diaz is entitled to one-half, or 8/16, of the retirement
insurance benefits while Berdin and their 7 children are entitled to the remaining one-half, or 8/16, each of them to receive an equal
share of 1/16.Dissatisfied, Berdin and her children filed a petition for mandamus with preliminary injunction in the CFI of Surigao, Southert Luzon Employee’s Association v. Golpeo - Insurance Beneficiaries
against the respondents praying that they be declared the legal heirs and exclusive beneficiaries of the retirement insurance. CFI
confirmed the action of GSIS. Hence the present appeal by herein petitioners-appellants, Basilia Berdin and her children contending 96 PHIL 83
that because the deceased failed to designate the beneficiaries in his retirement insurance, they being beneficiaries named in the life Facts:
insurance should automatically be considered the beneficiaries to receive the retirement insurance benefits, to the exclusion of Diaz. > SLEA is composed of laborers and employees of the LTBC and BTC (now BLTB Co.), and one of its
ISSUES: purposes is mutual aid of its members and their dependents in case of death.
(a) Is the contention tenable?(b) To whom should this retirement insurance benefits be paid? > Roman Concepcion was a member until his death in 1950.
RULING: > In 1949, SLEA adopted a resolution providing that: A member may, if he chooses, put down his common
A.NO. The contention of appellants is untenable.When Consuegra designated his beneficiaries in his life insurance he could not law wife and/or children he had with her as his beneficiaries; and such person so named by the member
have intended those beneficiaries of his lifeinsurance as also the beneficiaries of his retirement insurance because the provisions on will be the sole persons to be recognized by SLEA regarding claims for condolence contributions.
retirement insurance under theGSIS came about only when Com. Act 186 was amended by Rep. Act 660 on June 16, 1951. > Roman listed as his beneficiaries Aquilina Maloles and their 4 children. After his death, SLEA was able
Hence, it cannot be said that to collect voluntary contribution from its members amounting to P2,205.
> Three sets of claimants to the amount presented themselves to the association namely:
Page 25 of 39
o Juanita Golpeo, legal wife, and her children Odessa, Karl Brian, and Trisha Angelie, as the remaining designated beneficiaries; and that it released
o Aquilina Maloles, the common law wife, and her children Odessa’s share as she was of age, but withheld the release of the shares of minors Karl Brian and Trisha
o Elsie Hicban, another common law wife of Roman, and her child. Angelie pending submission of letters of guardianship. Insular alleged that the complaint or petition failed
> SLEA then filed an action for interpleader against the 3 conflicting claimants. to state a cause of action insofar as it sought to declare as void the designation of Eva as beneficiary,
> Trial court rendered a decision declaring Maloles and her children the sole beneficiaries of the amount because Loreto revoked her designation as such in Policy No. A001544070 and it disqualified her in Policy
citing Del Val v. Del Val. No. A001693029; and insofar as it sought to declare as inofficious the shares of Odessa, Karl Brian, and
> Only Golpeo appealed. She argues that: Trisha Angelie, considering that no settlement of Loreto’s estate had been filed nor had the respective
> The insurance code does not apply since the association is not an insurance company but a mutual shares of the heirs been determined. Insular further claimed that it was bound to honor the insurance
benefit association. policies designating the children of Loreto with Eva as beneficiaries pursuant to Section 53 of the
> The stipulation between SLEA and Roman was void for being contrary to law, public morals and public Insurance Code. Grepalife alleged that Eva was not designated as an insurance policy beneficiary; that the
policy, pursuant to Art. 739 of the CC ( donations between persons guilty of concubinage at the time of claims filed by Odessa, Karl Brian, and Trisha Angelie were denied because Loreto was ineligible for
donation are void) insurance due to a misrepresentation in his application form that he was born on December 10, 1936 and,
thus, not more than 65 years old when he signed it in September 2001; that the case was premature, there
Issue: being no claim filed by the legitimate family of Loreto; and that the law on succession does
not apply where the designation of insurance beneficiaries is clear.
Whether or not Golpeo, the legal wife is entitled to the amount. ISSUE:
Whether or not illegitimate children can be beneficiaries in an insurance contract.
Held: RULING:
NO. Yes. Section 53 of the Insurance Code states that the insurance proceeds shall be applied exclusively to the
First of all, the lower court did not consider the association as a regular insurance company, but merely proper interest of the person in whose name or for whose benefit it is made unless otherwise specified in
ruled that the death benefit in question is analogous to insurance. Besides, even the Administrative Code the policy. Pursuant thereto, it is obvious that the only persons entitled to claim the insurance proceeds are
describes a mutual benefit company as one which provides any method of life insurance among its either the insured, if still alive; or the beneficiary, if the insured is already deceased, upon the maturation
members out of dues or assessments collected from its membership. of the policy.The exception to this rule is a situation where the insurance contract was intended to benefit
third persons who are not parties to the same in the form of favorable stipulations or indemnity. In such a
Secondly, without considering the intimation in the brief for Maloles that Golpeo, by her silence and case, third parties may directly sue and claim from the insurer.
actions had acquiesced in the illicit relations between her husband and Maloles, Golpeo’s argument would Petitioners are third parties to the insurance contracts with Insular and Grepalife and, thus, are not entitled
certainly NOT apply to the children of Maloles likewise named beneficiaries by the deceased. As a matter to the proceeds thereof. Accordingly, respondents Insular and Grepalife have no legal obligation to turn
of fact, the NCC recognizes certain successional rights of illegitimate children. over the insurance proceeds to petitioners. The revocation of Eva as a beneficiary in one policy and
her disqualification as such in another are of no moment considering that the designation of the illegitimate
children as beneficiaries in Loreto’s insurance policies remains valid. Because no legal
Heirs of Maramag v. Maramag proscription exists in naming as beneficiaries the children of illicit relationships by the insured, the shares
G.R. No. 181132 , June 5, 2009 of Eva in the insurance proceeds, whether forfeited by the court in view of the prohibition
FACTS: on donations under Article 739 of the Civil Code or by the insurers themselves for reasons based on the
The case stems from a petition filed against respondents with the RTC for revocation and/or reduction of insurance contracts, must be awarded to the said illegitimate children, the designated beneficiaries, to
insurance proceeds for being void and/or inofficious. The petition alleged that: (1) petitioners were the the exclusion of petitioners. It is only in cases where the insured has not designated any beneficiary, or
legitimate wife and children of Loreto Maramag (Loreto), while respondents were Loreto’s illegitimate when the designated beneficiary is disqualified by law to receive the proceeds, that the insurance policy
family; (2) Eva de Guzman Maramag (Eva) was a concubine of Loreto and a suspect in the killing of proceeds shall redound to the benefit of the estate of the insured.
the latter, thus, she is disqualified to receive any proceeds from his insurance policies from Insular Life
Assurance Company, Ltd. (Insular) and Great Pacific Life Assurance Corporation (Grepalife) (3) the
illegitimate children of Loreto—Odessa, Karl Brian, and Trisha Angelie—were entitled only to one-half of
the legitime of the legitimate children, thus, the proceeds released to Odessa and those to be released to Philamcare v. CA- Health Care Agreement
Karl Brian and Trisha Angelie were inofficious and should be reduced; and (4) petitioners could not
be deprived of their legitimes, which should be satisfied first. Insular admitted that Loreto misrepresented 379 SCRA 356 (2002)
Eva as his legitimate wife and Odessa, Karl Brian, and Trisha Angelie as his legitimate children, and that Facts:
they filed their claims for the insurance proceeds of the insurance policies; that when it ascertained that Eva > Ernani Trinos, applied for a health care coverage with Philamcare. In the standard application form, he
was not the legal wife of Loreto, it disqualified her as a beneficiary and divided the proceeds among answered NO to the following question: “Have you or any of your family members ever consulted or been
Page 26 of 39
treated for high blood pressure, heart trouble, diabetes, cancer, liver disease, asthma or peptic ulcer? (If Failure to disclose or misrepresentation of any material information by the member in the application or
Yes, give details)” medical examination, whether intentional or unintentional, shall automatically invalidate the Agreement
> The application was approved for a period of one year from March 1, 1988 to March 1, 1989. He was a from the very beginning and liability of Philamcare shall be limited to return of all Membership Fees paid.
issued Health Care Agreement, and under such, he was entitled to avail of hospitalization benefits, whether An undisclosed or misrepresented information is deemed material if its revelation would have resulted in
ordinary or emergency, listed therein. He was also entitled to avail of "out-patient benefits" such as annual the declination of the applicant by Philamcare or the assessment of a higher Membership Fee for the
physical examinations, preventive health care and other out-patient services. benefit or benefits applied for.
> Upon the termination of the agreement, the same was extended for another year from March 1, 1989 to
March 1, 1990, then from March 1, 1990 to June 1, 1990. The amount of coverage was increased to a The answer assailed by petitioner was in response to the question relating to the medical history of the
maximum sum of P75,000.00 per disability. applicant. This largely depends on opinion rather than fact, especially coming from respondent's husband
> During the period of his coverage, Ernani suffered a heart attack and was confined at the Manila Medical who was not a medical doctor. Where matters of opinion or judgment are called for, answers made in good
Center (MMC) for one month beginning March 9, 1990. faith and without intent to deceive will not avoid a policy even though they are untrue. Thus,
> While her husband was in the hospital, Julita tried to claim the benefits under the health care agreement. (A)lthough false, a representation of the expectation, intention, belief, opinion, or judgment of the insured
However, Philamcare denied her claim saying that the Health Care Agreement was void. will not avoid the policy if there is no actual fraud in inducing the acceptance of the risk, or its acceptance
> According to Philamcare, there was concealment regarding Ernani's medical history. at a lower rate of premium, and this is likewise the rule although the statement is material to the risk, if the
Doctors at the MMC allegedly discovered at the time of Ernani's confinement that he was hypertensive, statement is obviously of the foregoing character, since in such case the insurer is not justified in relying
diabetic and asthmatic, contrary to his answer in the application form. upon such statement, but is obligated to make further inquiry. There is a clear distinction between such a
> Julita had no choice but to pay the hospitalization expenses herself, amounting to about P76,000.00 case and one in which the insured is fraudulently and intentionally states to be true, as a matter of
> After her husband was discharged from the MMC, he was attended by a physical therapist at home. expectation or belief, that which he then knows, to be actually untrue, or the impossibility of which is
Later, he was admitted at the Chinese General Hospital (CGH). Due to financial difficulties, Julita brought shown by the facts within his knowledge, since in such case the intent to deceive the insurer is obvious and
her husband home again. In the morning of April 13, 1990, Ernani had fever and was feeling very weak. amounts to actual fraud.
Julita was constrained to bring him back to the CGH where he died on the same day. The fraudulent intent on the part of the insured must be established to warrant rescission of the insurance
> Julita instituted, an action for damages against Philamcare. She asked for reimbursement of her expenses contract. Concealment as a defense for the health care provider or insurer to avoid liability is an affirmative
plus moral damages and attorney's fees. RTC decided in favor of Julita. CA affirmed. defense and the duty to establish such defense by satisfactory and convincing evidence rests upon the
provider or insurer. In any case, with or without the authority to investigate, petitioner is liable for claims
Issues and Resolutions: made under the contract. Having assumed a responsibility under the agreement, petitioner is bound to
Philamcare brought the instant petition for review, raising the primary argument that a health care answer the same to the extent agreed upon. In the end, the liability of the health care provider attaches once
agreement is not an insurance contract; hence the "incontestability clause" under the Insurance Code Title the member is hospitalized for the disease or injury covered by the agreement or whenever he avails of the
6, Sec. 48 does not apply. covered benefits which he has prepaid.
Under Section 27 of the Insurance Code, "a concealment entitles the injured party to rescind a contract of
SC held that in the case at bar, the insurable interest of respondent's husband in obtaining the health care insurance." The right to rescind should be exercised previous to the commencement of an action on the
agreement was his own health. The health care agreement was in the nature of non-life insurance, which is contract. In this case, no rescission was made. Besides, the cancellation of health care agreements as in
primarily a contract of indemnity. Once the member incurs hospital, medical or any other expense arising insurance policies require the concurrence of the following conditions:
from sickness, injury or other stipulated contingent, the health care provider must pay for the same to the Prior notice of cancellation to insured;
extent agreed upon under the contract. Notice must be based on the occurrence after effective date of the policy of one or more of the grounds
mentioned;
Under the title Claim procedures of expenses, Philamcare. had 12 mos from the date of issuance of the Must be in writing, mailed or delivered to the insured at the address shown in the policy;
Agreement within which to contest the membership of the patient if he had previous ailment of asthma, and Must state the grounds relied upon provided in Section 64 of the Insurance Code and upon request of
six months from the issuance of the agreement if the patient was sick of diabetes or hypertension. The insured, to furnish facts on which cancellation is based.
periods having expired, the defense of concealment or misrepresentation no longer lie.
Petitioner argues that respondent's husband concealed a material fact in his application. It appears that in None of the above pre-conditions was fulfilled in this case. When the terms of insurance contract contain
the application for health coverage, petitioners required respondent's husband to sign an express limitations on liability, courts should construe them in such a way as to preclude the insurer from non-
authorization for any person, organization or entity that has any record or knowledge of his health to compliance with his obligation. Being a contract of adhesion, the terms of an insurance contract are to be
furnish any and all information relative to any hospitalization, consultation, treatment or any other medical construed strictly against the party which prepared the contract — the insurer. By reason of the exclusive
advice or examination. control of the insurance company over the terms and phraseology of the insurance contract, ambiguity must
be strictly interpreted against the insurer and liberally in favor of the insured, especially to avoid forfeiture.
Philamcare cannot rely on the stipulation regarding "Invalidation of agreement" which reads: This is equally applicable to Health Care Agreements. The phraseology used in medical or hospital service
Page 27 of 39
contracts, such as the one at bar, must be liberally construed in favor of the subscriber, and if doubtful or Lessons Applicable: Insured Outlives Policy (Insurance)
reasonably susceptible of two interpretations the construction conferring coverage is to be adopted, and Laws Applicable:
exclusionary clauses of doubtful import should be strictly construed against the provider.

Nario V. Philippine American Life Insurance Co. Of Canada (1967) FACTS:


G.R. No. L-22796 June 26, 1967 West Coast Life Insurance Company issued 2 policies of insurance on the life of Esperanza J. Villanueva:
Lessons Applicable: Irrevocable Designation (Insurance) 2,000 php - maturing on April 1, 1943
if living, on the 1st day of April 1943 - to insured
FACTS: upon death during the continuance of this policy - to the beneficiary Bartolome Villanueva, father of the
June 12, 1959: Philippine American Life Insurance Co. issued a life insurance to Mrs. Alejandra Santos- insured, with right on the part of the insured to change the beneficiary
Mario a life insurance policy under a 20-year endowment plan, with a face value of P5,000 designating her 1940: Bartolome Villanueva died, Mariano J. Villanueva duly substituted as beneficiary, a brother of the
husband Delfin Nario and their unemancipation son Ernesto Nario, as her irrevocable beneficiaries insured
June, 1963: She submitted her loan application to the life insurance co. with signature of her husband in 3,000 php - maturing on March 31, 1943
two capacities: Esperanza J. Villanueva survived the insurance period, for she died only on October 15, 1944, without,
irrevocable beneficiaries however, collecting the insurance proceeds.
father-guardian of minor irrevocable beneficiary Ernesto CFI: estate of the insured Esperanza is entitled to the insurance proceeds
Insurance Co. denied asking that the legal guardian must be authorized by the court in a competent ISSUE: W/N the estate of insured Esperanza should be entitled to the insurance proceeds since she outlived
guardianship proceeding the insurance policy
Upon denial, she opted to surrender her insurance policy in exchange of its cash surrender value of P520
but it was also denied on the same ground
September 10, 1963: Mrs. Alejandra Santos-Nario and her husband, Delfin Nario, brought suit against the HELD: YES. appealed order is, therefore, hereby affirmed
Philippine American Life Insurance Co To sustain the beneficiary's claim would be altogether eliminate from the policies the condition that the
RTC: favored the insurance company insurer "agrees to pay . . . to the insured hereunder, if living
CA: vested interest or right of the beneficiaries in the policy should be measured on its full face value and Upon the insured's death, within the period, the beneficiary will take, as against the personal representative
not on its cash surrender value, for in case of death of the insured, said beneficiaries are paid on the basis of or the assignee of the insured. Upon the other hand, if the insured survives the endowment period, the
its face value and in case the insured should discontinue paying premiums, the beneficiaries may continue benefits are payable to him or to his assignee, notwithstanding a beneficiary is designated in the policy
paying it and are entitled to automatic extended term or paid-up insurance options, etc. and that said vested
right under the policy cannot be divisible at any given time. policy loan and surrender of policy constitute Villanueva v. Oro - Insurance Proceeds
acts of disposition or alienation of property rights and not merely of management or administration because 81 PHIL 464
they involve the incurring or termination of contractual obligations Facts:
ISSUE: W/N parents as guardians can enter into transactions for the benefit of minor irrevocable > West Coast Life Insurance Company issued two policies of insurance on the life of Esperanza
beneficiaries. Villanueva, one for 2T, maturing April 1, 1943; and other for 3T maturing Mar. 31, 1943.
> In both policies, West agreed to pay 2T either to Esperanza if still living on Apr 1, 1943; or to
beneficiary Bartolome Villanueva, or the father of the insured immediately upon receipt of the proof of
HELD: NO. Affirmed. death of Esperanza.
SEC. 7. Parents as guardians. — When the property of the child under parental authority is worth two > The policy also gave her the right to change the beneficiary.
thousand pesos or less, the father or the mother, without the necessity of court appointment, shall be his > In 1940, Bartolome died, and he was substituted as beneficiary under the policies by Mariano,
legal guardian. When the property of the child is worth more than two thousand pesos, the father or the Esparanza’s brother.
mother shall be considered guardian of the child's property, with the duties and obligations of guardians > Esperanza died in 1944 without having collected the insurance proceeds. Adverse claims for the
under these rules, and shall file the petition required by Section 2 hereof. For good reasons the court may, proceeds were presented by the estate of Esperanza on one hand and by Mariano on the other.
however, appoint another suitable person. > CFI held that the estate of Esperanza was entitled to the proceeds to the exclusion of the beneficiary.
even if worth less than P2,000 parent's authority over the estate of the ward as a legal-guardian would not
extend to acts of encumbrance or disposition, as distinguished from acts of management or administration. Issue:
Whether or not the beneficiary is entitled to the proceeds.
Villanueva V. Oro (1948)
G.R. No. L-2227 August 31, 1948
Page 28 of 39
Held: he will be benefited in its continued existence or suffer a direct pecuniary loss from its destruction or
NO. injury. The test in determining insurable interest in property is whether one will derive pecuniary benefit or
Under the policies, the insurer obligated itself to pay the insurance proceeds to: (1) the insured if the latter advantage from its preservation, or will suffer pecuniary loss or damage from its destruction, termination or
lived on the dates of maturity; or (2) the beneficiary if the insured died during the continuance of the injury by the happening of the event insured against.
policies. The first contingency excludes the second, and vice versa. In other words, as the insured
Esperanza was living on April 1 and March 31, 1943, the proceeds are payable exclusively to her or to her Here Harvardian was not only in possession of the building but was in fact using the same for several years
estate unless she had before her death otherwise assigned the matured policies. with the knowledge and consent of Ildefonso Yap. It is reasonably fair to assume that had the building not
been burned, Harvardian would have been allowed the continued use of the same as the site of its operation
The beneficiary could be entitled to said proceeds only in default of the first contingency. To sustain the as an educational institution. Harvardian therefore would have been directly benefited by the preservation
beneficiary’s claim would be to altogether eliminate from the policies the condition that the insurer “agrees of the property, and certainly suffered a pecuniary loss by its being burned.
to pay to the insured if living.”

This conclusion tallies with American Authorities who say that: The interest of the insured in the proceeds Gaisano Cagayan, Inc. V. Insurance Company Of North America (2006)
of the insurance depends upon his survival of the expiration of the endowment period. Upon the insured’s
death, within the period, the beneficiary will take, as against the personal representatives the endowment G.R. No. 147839 June 8, 2006
period, the benefits are payable to him or to his assignee, notwithstanding a beneficiary is designated in the
policy. (AmJur and Couch Cyclopedia of Insurance Law) Lessons Applicable: Existing Interest (Insurance)
Laws Applicable: Article 1504,Article 1263, Article 2207 of the Civil Code, Section 13 of Insurance Code
WEEK 6

Harvardian Colleges v. Country Bankers Insurance Corp. FACTS:


1 CARA 2 Intercapitol Marketing Corporation (IMC) is the maker of Wrangler Blue Jeans. while Levi Strauss (Phils.)
Inc. (LSPI) is the local distributor of products bearing trademarks owned by Levi Strauss & Co
Facts: IMC and LSPI separately obtained from Insurance Company of North America fire insurance policies for
> Harvardian is a family corporation, the stockholders of which are Ildefonso Yap, Virginia King Yap and their book debt endorsements related to their ready-made clothing materials which have been sold or
their children. delivered to various customers and dealers of the Insured anywhere in the Philippines which are unpaid 45
> Prior to Aug. 9, 1979, an agent of Country Bankers proposed to Harvardian to insure its school building. days after the time of the loss
Although at first reluctant, Harvardian agreed. February 25, 1991: Gaisano Superstore Complex in Cagayan de Oro City, owned by Gaisano Cagayan,
> Country Banks sent an inspector to inspect the school building and agreed to insure the same for Inc., containing the ready-made clothing materials sold and delivered by IMC and LSPI was consumed by
P500,000 for which Harvardian paid an annual premium of P2,500. fire.
> On Aug. 9, 1979, Country Bankers issued to Harvardian a fire insurance policy. On March 12, 1980, (39 February 4, 1992: Insurance Company of North America filed a complaint for damages against Gaisano
days before I was born… hehehehe )during the effectivity of said insurance policy, the insured property was Cagayan, Inc. alleges that IMC and LSPI filed their claims under their respective fire insurance policies
totally burned rendering it a total loss. which it paid thus it was subrogated to their rights
> A claim was made by plaintiff upon defendant but defendant denied it contending that plaintiff had no Gaisano Cagayan, Inc: not be held liable because it was destroyed due to fortuities event or force majeure
insurable interest over the building constructed on the piece of land in the name of the late Ildefonso Yap as RTC: IMC and LSPI retained ownership of the delivered goods until fully paid, it must bear the loss (res
owner. perit domino)
> It was contended that both the lot and the building were owned by Ildefonso Yap and NOT by the CA: Reversed - sales invoices is an exception under Article 1504 (1) of the Civil Code to res perit domino
Harvardian Colleges. ISSUE: W/N Insurance Company of North America can claim against Gaisano Cagayan for the debt that
was isnured
Issue:
Whether or not Harvardian colleges has a right to the proceeds. HELD: YES. petition is partly GRANTED. order to pay P535,613 is DELETED
insurance policy is clear that the subject of the insurance is the book debts and NOT goods sold and
Held: delivered to the customers and dealers of the insured
Harvardian has a right to the proceeds.
Regardless of the nature of the title of the insured or even if he did not have title to the property insured, the
contract of fire insurance should still be upheld if his interest in or his relation to the property is such that
Page 29 of 39
ART. 1504. Unless otherwise agreed, the goods remain at the seller's risk until the ownership therein is IMC and Levi Strauss (Phils.) Inc. (LSPI) separately obtained from respondent fire insurance policies with
transferred to the buyer, but when the ownership therein is transferred to the buyer the goods are at the book debt endorsements. The insurance policies provide for coverage on "book debts in connection with
buyer's risk whether actual delivery has been made or not, except that: ready-made clothing materials which have been sold or delivered to various customers and dealers of the
Insured anywhere in the Philippines."
(1) Where delivery of the goods has been made to the buyer or to a bailee for the buyer, in pursuance of the The policies defined book debts as the "unpaid account still appearing in the Book of Account of the
contract and the ownership in the goods has been retained by the seller merely to secure performance by the Insured 45 days after the time of the loss covered under this Policy." The policies also provide for the
buyer of his obligations under the contract, the goods are at the buyer's risk from the time of such delivery; following conditions:
IMC and LSPI did not lose complete interest over the goods. They have an insurable interest until full 1. Warranted that the Company shall not be liable for any unpaid account in respect of the merchandise sold
payment of the value of the delivered goods. Unlike the civil law concept of res perit domino, where and delivered by the Insured which are outstanding at the date of loss for a period in excess of six (6)
ownership is the basis for consideration of who bears the risk of loss, in property insurance, one's interest is months from the date of the covering invoice or actual delivery of the merchandise whichever shall first
not determined by concept of title, but whether insured has substantial economic interest in the property occur.
Section 13 of our Insurance Code defines insurable interest as "every interest in property, whether real or 2. Warranted that the Insured shall submit to the Company within twelve (12) days after the close of
personal, or any relation thereto, or liability in respect thereof, of such nature that a contemplated peril every calendar month all amount shown in their books of accounts as unpaid and thus become receivable
might directly damnify the insured." Parenthetically, under Section 14 of the same Code, an insurable item from their customers and dealers.
interest in property may consist in: (a) an existing interest; (b) an inchoate interest founded on existing Gaisano is a customer and dealer of the products of IMC and LSPI. On February 25, 1991, the Gaisano
interest; or (c) an expectancy, coupled with an existing interest in that out of which the expectancy arises. Superstore Complex in Cagayan de Oro City, owned by petitioner, was consumed by fire. Included in the
Anyone has an insurable interest in property who derives a benefit from its existence or would suffer loss items lost or destroyed in the fire were stocks of ready-made clothing materials sold and delivered by IMC
from its destruction. and LSPI.
it is sufficient that the insured is so situated with reference to the property that he would be liable to loss Insurance of America filed a complaint for damages against Gaisano. It alleges that IMC and LSPI were
should it be injured or destroyed by the peril against which it is insured paid for their claims and that the unpaid accounts of petitioner on the sale and delivery of ready-made
an insurable interest in property does not necessarily imply a property interest in, or a lien upon, or clothing materials with IMC was P2,119,205.00 while with LSPI it was P535,613.00.
possession of, the subject The RTC rendered its decision dismissing Insurance's complaint. It held that the fire was purely accidental;
matter of the insurance, and neither the title nor a beneficial interest is requisite to the existence of such an that the cause of the fire was not attributable to the negligence of the petitioner. Also, it said that IMC and
interest LSPI retained ownership of the delivered goods and must bear the loss.
insurance in this case is not for loss of goods by fire but for petitioner's accounts with IMC and LSPI that The CA rendered its decision and set aside the decision of the RTC. It ordered Gaisano to pay Insurance the
remained unpaid 45 days after the fire - obligation is pecuniary in nature P 2 million and the P 500,000 the latter paid to IMC and Levi Strauss.
obligor should be held exempt from liability when the loss occurs thru a fortuitous event only holds true Hence this petition.
when the obligation consists in the delivery of a determinate thing and there is no stipulation holding him
liable even in case of fortuitous event Issues:
Article 1263 of the Civil Code in an obligation to deliver a generic thing, the loss or destruction of anything 1. WON the CA erred in construing a fire insurance policy on book debts as one covering the unpaid
of the same kind does not extinguish the obligation (Genus nunquan perit) accounts of IMC and LSPI since such insurance applies to loss of the ready-made clothing materials sold
The subrogation receipt, by itself, is sufficient to establish not only the relationship of respondent as insurer and delivered to petitioner
and IMC as the insured, but also the amount paid to settle the insurance claim 2. WON IMC bears the risk of loss because it expressly reserved ownership of the goods by stipulating in
Art. 2207. If the plaintiff's property has been insured, and he has received indemnity from the insurance the sales invoices that "[i]t is further agreed that merely for purpose of securing the payment of the
company for the injury or loss arising out of the wrong or breach of contract complained of, the insurance purchase price the above described merchandise remains the property of the vendor until the purchase price
company shall be subrogated to the rights of the insured against the wrongdoer or the person who has thereof is fully paid."
violated the contract. 3. WON petitioner is liable for the unpaid accounts
As to LSPI, no subrogation receipt was offered in evidence. 4. WON it has been established that petitioner has outstanding accounts with IMC and LSPI.
Failure to substantiate the claim of subrogation is fatal to petitioner's case for recovery of the amount of
P535,613 Held: No. Yes. Yes. Yes but account with LSPI unsubstantiated. Petition partly granted.

Gaisano v Insurance G.R. No. 147839 June 8, 2006 Ratio:


J. Martinez 1. Nowhere is it provided in the questioned insurance policies that the subject of the insurance is the goods
sold and delivered to the customers and dealers of the insured.
Facts: Thus, what were insured against were the accounts of IMC and LSPI with petitioner which remained
unpaid 45 days after the loss through fire, and not the loss or destruction of the goods delivered.
Page 30 of 39
2. The present case clearly falls under paragraph (1), Article 1504 of the Civil Code: As to LSPI, respondent failed to present sufficient evidence to prove its cause of action. There was no
ART. 1504. Unless otherwise agreed, the goods remain at the seller's risk until the ownership therein is evidence that respondent has been subrogated to any right which LSPI may have against petitioner. Failure
transferred to the buyer, but when the ownership therein is transferred to the buyer the goods are at the to substantiate the claim of subrogation is fatal to petitioner's case for recovery of P535,613.00.
buyer's risk whether actual delivery has been made or not, except that:
(1) Where delivery of the goods has been made to the buyer or to a bailee for the buyer, in pursuance of the Harding v Commerical Union August 10, 1918 G.R. No. L-12707
contract and the ownership in the goods has been retained by the seller merely to secure performance by the J. Fisher
buyer of his obligations under the contract, the goods are at the buyer's risk from the time of such delivery
Thus, when the seller retains ownership only to insure that the buyer will pay its debt, the risk of loss is Facts:
borne by the buyer. Petitioner bears the risk of loss of the goods delivered. Smith Bell insured Mrs. Hardings’ Studebaker car for a premium of Php 150. It was insured for Php 3,000,
IMC and LSPI had an insurable interest until full payment of the value of the delivered goods. Unlike the the value of the car. The car was destroyed by fire. Mrs. Harding furnished the defendant the proofs of her
civil law concept of res perit domino, where ownership is the basis for consideration of who bears the risk loss, but the company didn’t pay. Evidence showed that Hermanos sold the automobile to Canson for
of loss, in property insurance, one's interest is not determined by concept of title, but whether insured has P3,200. Canson then sold the car to Harding for Php 1,500. The car was then sold for P2,000. It was then
substantial economic interest in the property. resold to Harding. He gave the car to his wife; Mrs. Henry E. Harding as a present. The automobile was
Section 13 of our Insurance Code defines insurable interest as "every interest in property, whether real or repaired and repainted at the Luneta Garage at P900.
personal, or any relation thereto, or liability in respect thereof, of such nature that a contemplated peril The company averred that they gave false information, particularly that on the price of the vehicle and the
might directly damnify the insured." Parenthetically, under Section 14 of the same Code, an insurable ownership of the car. Hence, they aimed to declare the policy void.
interest in property may consist in: (a) an existing interest; (b) an inchoate interest founded on existing The trial court found that there was no fraud.
interest; or (c) an expectancy, coupled with an existing interest in that out of which the expectancy arises. This was an action by plaintiffs to recover from defendant the sum of P3,000 and interest, alleged to be due
Anyone has an insurable interest in property who derives a benefit from its existence or would suffer loss under the terms of a policy of insurance. The trial court gave plaintiffs judgment for the amount demanded,
from its destruction. Indeed, a vendor or seller retains an insurable interest in the property sold so long as with interest and costs, and from that decision the defendant appealed.
he has any interest therein, in other words, so long as he would suffer by its destruction, as where he has a
vendor's lien. In this case, the insurable interest of IMC and LSPI pertain to the unpaid accounts appearing Issue: Was the valuation of the car for P3000 done fraudulently, thereby making the policy void?
in their Books of Account 45 days after the time of the loss covered by the policies.
3. Petitioner's argument that it is not liable because the fire is a fortuitous event under Article 117432 of the Held: No.
Civil Code is misplaced. As held earlier, petitioner bears the loss under Article 1504 (1) of the Civil Code.
Moreover, it must be stressed that the insurance in this case is not for loss of goods by fire but for Ratio:
petitioner's accounts with IMC and LSPI that remained unpaid 45 days after the fire. Accordingly, The policy stated that
petitioner's obligation is for the payment of money. As correctly stated by the CA, where the obligation “That during the period above set forth and during any period for which the company may agree to renew
consists in the payment of money, the failure of the debtor to make the payment even by reason of a this policy the company will subject to the exception and conditions contained herein or endorsed hereon
fortuitous event shall not relieve him of his liability. The rationale for this is that the rule that an obligor indemnify the insured against loss of or damage to any motor car described in the schedule by whatever
should be held exempt from liability when the loss occurs thru a fortuitous event only holds true when the cause such loss or damage may be occasioned and will further indemnify the insured up to the value of the
obligation consists in the delivery of a determinate thing and there is no stipulation holding him liable even car or P3,000 whichever is the greater against any claim at common law made by any person for loss of life
in case of fortuitous event. It does not apply when the obligation is pecuniary in nature. or for accidental bodily injury or damage to property caused by the said motor car including law costs
Under Article 1263 of the Civil Code, "[i]n an obligation to deliver a generic thing, the loss or destruction payable in connection with such claim when incurred with the consent of the company.”
of anything of the same kind does not extinguish the obligation." This rule is based on the principle that the Defendant contends that the statement regarding the cost of the automobile was a warranty, that the
genus of a thing can never perish. An obligation to pay money is generic; therefore, it is not excused by statement was false, and that, therefore, the policy never attached to the risk.
fortuitous loss of any specific property of the debtor. The automobile had in fact cost more than the amount mentioned. The court below found, and the evidence
4. With respect to IMC, the respondent has adequately established its claim. The P 3 m claim has been shows, that the automobile was bought by plaintiff’s husband a few weeks before the issuance of the policy
proven. The subrogation receipt, by itself, is sufficient to establish not only the relationship of respondent in question for the sum of P2,800, and that between that time and the issuance of the policy some P900 was
as insurer and IMC as the insured, but also the amount paid to settle the insurance claim. The right of spent upon it in repairs and repainting. The mechanic who testified told that the automobile was practically
subrogation accrues simply upon payment by the insurance company of the insurance claim Respondent's as good as new at the time the insurance was effected.
action against petitioner is squarely sanctioned by Article 2207 of the Civil Code which provides: The amount stated was less than the actual outlay which the automobile represented to Mr. Harding,
Art. 2207. If the plaintiff's property has been insured, and he has received indemnity from the insurance including repairs, when the insurance policy was issued. It would be unfair to hold the policy void simply
company for the injury or loss arising out of the wrong or breach of contract complained of, the insurance because the outlay represented by the automobile was made by the plaintiff’s husband and not by his wife,
company shall be subrogated to the rights of the insured against the wrongdoer or the person who has to whom he had given the automobile.
violated the contract.
Page 31 of 39
The trial court found that Mrs. Harding, in fixing the value of the automobile at P3,000, acted upon After its destruction, he collected P3,600 from the insurance company, having paid in premiums the
information given her by her husband and by Mr. Server, the manager of the Luneta Garage. She merely
repeated the information which had been given her by her husband, and at the same time disclosed to sum of P301.50
defendant’s agent the source of her information. There is no evidence to sustain the contention that this  Lampano filed a complaint against Barreto and Jose alleging that Jose in a verbal agreement told
communication was made in bad faith.
Under these circumstances, we do not think that the facts stated in the proposal can be held as a warranty of her that the policy will be delivered to her so she should collected P3,600 from each of them
the insured, even if it should have been shown that they were incorrect in the absence of proof of willful  RTC: favored Jose ordering Barreto to pay him P1,298.50 and offsetting the P2,000
misstatement. Under such circumstance, the proposal is to be regarded as the act of the insurer and not of
the insured.  Barreto alone appealed
The defendant, upon the information given by plaintiff, and after an inspection of the automobile by its ISSUE: W/N Barreto had insurable interest in the house and could insure it for his it for his own protection
examiner, having agreed that it was worth P3,000, is bound by this valuation in the absence of fraud on the
part of the insured. All statements of value are, of necessity, to a large extent matters of opinion, and it
would be outrageous to hold that the validity of all valued policies must depend upon the absolute HELD: YES. reversed and Barretto is absolved
correctness of such estimated value.  Where different persons have different interests in the same property, the insurance taken by one
Supreme Court v First National- The ordinary test of the value of property is the price it will commend in
the market if offered for sale. But that test cannot, in the very nature of the case, be applied at the time in his own right and in his own interest does not in any way insure to the benefit of another
application is made for insurance. Men may honestly differ about the value of property, or as to what it will  A contract of insurance made for the insurer's (insured) indemnity only, as where there is no
bring in the market; and such differences are often very marked among those whose special business it is to
buy and sell property of all kinds. agreement, express or implied, that it shall be for the benefit of a third person, does not attach to or run
The assured could do no more than estimate such value; and that, it seems, was all that he was required to with the title to the insured property on a transfer thereof personal as between the insurer and the
do in this case. His duty was to deal fairly with the Company in making such estimate. insured.
Section 163 of the Insurance Law (Act No. 2427) provides that “the effect of a valuation in a policy of fire
insurance is the same as in a policy of marine insurance.”  Barretto had an insurable interest in the house. He construed the building, furnishing all the
By the terms of section 149 of the Act cited, the valuation in a policy of marine insurance is conclusive if materials and supplies, and insured it after it had been completed
the insured had an insurable interest and was not guilty of fraud.
The valuation of the automobile, for the purposes of the insurance, is binding upon the defendant
corporation. Insurance Case Digest: Ong Lim Sing V. FEB Leasing And Finance Corp. (2007)

Insurance Case Digest: Lampano V. Jose (1915) G.R. No. 168115 June 8, 2007

G.R. No. L-9401 March 30, 1915 Lessons Applicable: Existing Interest (Insurance)

Lessons Applicable: Existing Interest (Insurance) Laws Applicable:


Laws Applicable: FACTS:
FACTS:
 FEB Leasing and Finance Corporation (FEB) leased equipment and motor vehicles to JVL Food
 Mariano R. Barretto, constructed a house for Placida A. Jose sold the house to Antonina Lampano Products with a monthly rental of P170,494
for P6,000  At the same date, Vicente Ong Lim Sing, Jr. (Lim) an executed an Individual Guaranty Agreement
with FEB to guarantee the prompt and faithful performance of the terms and conditions of the lease
 The house was destroyed by fire during which Lampano still owed Jose P2,000 as evidenced by a agreement
promissory note. Jose also owed Barretto P2,000 for the construction.  JVL defaulted in the payment of the monthly rentals resulting to arrears of P3,414,468.75 and
refused to pay despite demands
 After the completion of the house and before it was destroyed, Mariano R. Barretto took out an
 FEB filed a complaint for damages and replevin against JVL, Lim and John Doe
insurance policy upon it in his own name, with the consent of Placida A. Jose, for the sum of P4,000.  JVL and Lim admitted the existence of the lease agreement but asserted that it is in reality
a sale of equipment on installment basis, with FEB acting as the financier

Page 32 of 39
 RTC: Sale on installment and the FEB elected full payment of the obligation so for the unreturned  fire insurance policy states: "that all insurancecovered under said policy, includes the 'rent
units and machineries the JVL and Lim are jointly and severally liable to pay or othersubject matter of insurance in respect of or inconnection with any building or any property
 CA: granted FEB appeal that it is a financial lease agreement under Republic Act (R.A.) No. 8556 contained in any building"
and ordered JVL and Lim jointly and severally to pay P3,414,468.75  June 5, 1949: the building premises was burned so Golangco requested
Trader’s Insurance to pay the insurance amount of 10,000 including the amount of rent P1,100
ISSUE: W/N JVL and Lim should jointly and severally be liable for the insured financial lease
monthly.
HELD: YES. CA affirmed.  Trader’s insurance refused to pay the insurance for the rent averring that Golangco has no
insurable interest
 contract of adhesion is as binding as any ordinary contract
 The Lease Contract with corresponding Lease Schedules with Delivery and Acceptance ISSUE: W/N Golangco has insurable interest on the rent of the building premises which may
Certificates is, in point of fact, a financial lease within the purview of R.A. No. 8556 lawfully/validly be subject of insurance?
 FEB leased the subject equipment and motor vehicles to JVL in consideration of a
monthly periodic payment of P170,494.00. The periodic payment by petitioner is sufficient to HELD: YES.
amortize at least 70% of the purchase price or acquisition cost of the said movables in accordance with
the Lease Schedules with Delivery and Acceptance Certificates.  Sec. 13 of the Insurance Code: Every interest in the property, whether real or personal, or any
 JVL entered into the lease contract with full knowledge of its terms and conditions. relation thereto, or liability in respect thereof of such nature that a contemplated peril might directly
 Lim, as a lessee, has an insurable interest in the equipment and motor vehicles leased. damnify the insured, is an insurable interest.
 In the financial lease agreement, FEB did not assume responsibility as to the quality,  Both at the time of the issuance of the policy and at the time of the fire,
merchantability, or capacity of the equipment. This stipulation provides that, in case of defect of any Golangco was in legal possession of the premises, collecting rentals from its occupant.
kind that will be found by the lessee in any of the equipment, recourse should be made to the  The argument of Trader’s Insurance that a policy of insurance must specify the interest of the
manufacturer. “The financial lessor, being a financing company, i.e., an extender of credit rather than insured in the property insured, if he is not the absolute owner thereof, is not meritorious because it
an ordinary equipment rental company, does not extend a warranty of the fitness of the equipment for was the Trader’s, not Golangco, who prepared that policy, and it cannot take advantage of its own acts
any particular use. Thus, the financial lessee was precisely in a position to enforce such warranty to plaintiff's detriment; and, in any case, this provisionwas substantially complied with by Golangco
directly against the supplier of the equipment and not against the financial lessor. We find nothing when he made a full and clear statement of his interests to Trader's manager.
contra legem or contrary to public policy in such a contractual arrangement  The contract between Lianco and the Archbishop only forbade Lianco from transferring
'his rights as LESSEE but the contracts Lianco made in favor of Kaw Eng Siand plaintiff Golangco did
Insurance Case Digest: Traders Insurance & Surety Co. V. Golangco, Et Al (1954) not transfer such rights; hence no written consent thereto was necessary. At worst, the contract would
be voidable, but not a void contract, at the option of the Archbishop and it does not appear that it was
G.R. No. L-6442 Sep 21, 1954 ever exercised
Lessons Applicable: Existing Interest (Insurance)
Laws Applicable: Sec. 13 of the Insurance Code Insurance Case Digest: Lopez V. Del Rosario And Quiogue (1922)

FACTS:
G.R. No. L-19189 November 27, 1922
 Tomas Lianco and the Archbishop entered into a contract of lease on a parcel of landowned by Lessons Applicable: Carrier or Depositary (Insurance)
church
 As lessee, Lianco erected a building on the leased portion of the church’s land. Laws Applicable:
 Lianco transferred ownership of this building to Kaw Eng
Si,who later transferred the same to Golangco.
 Transfers were made without the consent of the Archbishop FACTS:
 The Archbishop filed an ejectment case against Lianco, who appears to be occupants of the
premises building with others paying rent to Golangco.  Benita Quiogue de V. del Rosario (Mrs. del Rosario), owner of a bonded warehouse where Froilan
 The right of Golangco to receive rent on the building was judicially recognized in a case
decided between Lianco and others occupying the premises pursuant to a compromise agreement. Lopez, holder or 14 waehouse receipts and Elias Zamora had their copra deposited
 The Archbishop did not exercise his option to question Golangco’s rights as lessee
 April 7,1949: Golangco applied for fire insurance with Trader’s Insurance and Surety Co.
Page 33 of 39
 Effect of transfer of thing insured (Insurance)
 The warehouse recipts states an insurance of 1% their declared value which can be increase or
Laws Applicable: sec. 16,sec. 19 (now sec. 20),sec. 50,sec.55 (now sec. 58) of the Insurance Code (all old
decrease by giving 1 month's notice in writing law)
 Lopez paid the insurance to May 18, 1920, but not thereafter FACTS:
 June 6, 1920: the warehouse was destroyed by fire. Only copra worth P49,985 was salvaged. At  In the contract of mortgage, the owner P.D. Dunn had agreed, at his own expense, to insure the
mortgaged property for its full value and to indorse the policies in such manner as to authorize the
that time Lopez was still liable for the storage and insurance of P315.90
Brewery Company to receive the proceeds in case of loss and to retain such part thereof as might be
 Mrs. Del Rosario submitted the insurance with the arbitrators and seems to have satisfied all of the necessary to satisfy the remainder then due upon the mortgage debt. Instead, however, of effecting the
persons who had copra stored in her warehouse, including the stockholders in the Compañia Coprera insurance himself Dunn authorized and requested the Brewery Company to procure insurance on the
property in the amount of P15,000 at Dunn's expense.
de Tayabas (whose stock she took over), with the exception of Froilan Lopez  San Miguel insured the property only as mortgagee.
 Ineffectual attempts by Mrs. Del Rosario to effect a compromise with Lopez first for P71,994,  Dunn sold the propert to Henry Harding. The insurance was not assigned by Dunn to Harding.
 When it was destroyed by fire, the two companies settled with San Miguelto the extent of the
later raised to P72,724, and finally reduced to P17,000, were made. But Lopez stubbornly contended, mortgage credit.
or, at least, his attorney contended for him, that he should receive not a centavo less than P88,595.43  RTC: Absolved the 2 companies from the difference. Henry Harding is not entitled to the
difference between the mortgage credit and the face value of the policies.
(from originally P107,990.40)
 Henry Harding appealed.
Issue: ISSUE:
1. W/N San Miguel has insurable interest as mortgagor only to the extent of the mortgage credit - YES
Whether or not Del Rosario acted as the agent of Lopez in taking out the insurance on the contents of the 2. W/N Harding has insurable interest as owner - NO
warehouse or whether she acted as the reinsurer of the copra.

HELD: affirmed
Held:
 section 19 of the Insurance Act:
She acted as the agent of Lopez.  a change of interest in any part of a thing insured unaccompanied by a corresponding
The agency can be deduced from the warehouse receipts, the insurance policies and the circumstances change of interest in the insurance, suspends the insurance to an equivalent extent, until the interest in
surrounding the transaction. Under any aspect, Del Rosario is liable. The law is that a policy effected by a the thing and the interest in the insurance are vested in the same person
bailee and covering by its terms in his own property and property held in trust, inures, in the event of loss,  section 55:
equally and proportionately to the benefit of all owners of the property insured. Even if one secured  the mere transfer of a thing insured does not transfer the policy, but suspends it until the
insurance covering his own goods and goods stored with him, and even if the owner of the stored goods did same person becomes the owner of both the policy and the thing insured
not request or know the insurance, and did not ratify it before the payment of the loss, it has been held by a  Undoubtedly these policies of insurance might have been so framed as to have been "payable to
reputable court that the warehouseman is liable to the owner of such stored goods for his share. the San Miguel Brewery, mortgagee, as its interest may appear, remainder to whomsoever, during the
continuance of the risk, may become the owner of the interest insured." (Sec 54, Act No. 2427.) Such a
In a case of contributing policies, adjustments of loss made by an expert or by a board of arbitrators may be clause would have proved an intention to insure the entire interest in the property, not merely the
submitted to the court NOT as evidence of the facts stated therein, or as obligatory, but for the purpose of insurable interest of the San Miguel Brewery, and would have shown exactly to whom the money, in
assisting the court in calculating the amount of liability. case of loss, should be paid. But the policies are not so written.
Insurance Case Digest: San Miguel Brewery V. Law Union And Rock Insurance Co. (1920)  The blame for the situation thus created rests, however, with the Brewery rather than with the
insurance companies, and there is nothing in the record to indicate that the insurance companies were
G.R. No. L-14300 January 19, 1920 requested to write insurance upon the insurable interest of the owner or intended to make themselves
liable to that extent
Lessons Applicable:  If by inadvertence, accident, or mistake the terms of the contract were not fully set forth in the
policy, the parties are entitled to have it reformed. But to justify the reformation of a contract, the
 Mortgagor (Insurance) proof must be of the most satisfactory character, and it must clearly appear that the contract failed to
 Measure of Insurable Interest (Insurance) express the real agreement between the parties
 Effect of Change of Interest in Thing Insured (Insurance)
Page 34 of 39
 In the case now before us the proof is entirely insufficient to authorize reformation.  The automatic assignment of the policy to CKS under the provision of the lease contract
previously quoted is void for being contrary to law and/or public policy. The proceeds of the fire
insurance policy thus rightfully belong to the spouses. The liability of the Cha spouses to CKS for
Insurance Case Digest: Cha V. CA (1997) violating their lease contract in that Cha spouses obtained a fire insurance policy over their own
merchandise, without the consent of CKS, is a separate and distinct issue which we do not resolve in
this case.
G.R. No. 124520 August 18, 1997
Insurance Case Digest: Tai Tong Chuache & Co. V. Insurance Commission (1988)

Lessons Applicable: Effect of Lack of Insurable Interest (Insurance)


G.R. No. L-55397 February 29, 1988
Laws Applicable: Sec. 17, Sec. 18, Sec. 25 of the Insurance Code Lessons Applicable: When Insurable Interest Must Exist (Insurance)

Laws Applicable:
FACTS:
FACTS:
 Spouses Nilo Cha and Stella Uy-Cha and CKS Development Corporation entered a 1 year lease
 Azucena Palomo bought a parcel of land and building from Rolando Gonzales and assumed a
contract with a stipulation not to insure against fire the chattels, merchandise, textiles, goods and
mortgage of the building in favor of S.S.S. which was insured with S.S.S. Accredited Group of
effects placed at any stall or store or space in the leased premises without first obtaining the written
Insurers
consent and approval of the lessor. But it insured against loss by fire their merchandise inside the
 April 19, 1975: Azucena Palomo obtained a loan from Tai Tong Chuache Inc. in the amount of
leased premises for P500,000 with the United Insurance Co., Inc. without the written consent of CKS
P100,000 and to secure it, the land and building was mortgaged
 On the day the lease contract was to expire, fire broke out inside the leased premises and CKS
 June 11, 1975: Pedro Palomo secured a Fire Insurance Policy covering the building for P50,000
learning that the spouses procured an insurance wrote to United to have the proceeds be paid directly
with Zenith Insurance Corporation
to them. But United refused so CKS filed against Spouses Cha and United.
 July 16, 1975: another Fire Insurance policy was procured from Philippine British Assurance
 RTC: United to pay CKS the amount of P335,063.11 and Spouses Cha to pay P50,000 as
Company, covering the same building for P50,000 and the contents thereof for P70,000
exemplary damages, P20,000 as attorney’s fees and costs of suit
 Before the occurrence of the peril insured against the Palomos had already paid their credit due the
 CA: deleted exemplary damages and attorney’s fees
 July 31, 1975: building and the contents were totally razed by fire
ISSUE: W/N the CKS has insurable interest because the spouses Cha violated the stipulation  Palomo was able to claim P41,546.79 from Philippine British Assurance Co., P11,877.14 from
HELD: NO. CA set aside. Awarding the proceeds to spouses Cha. Zenith Insurance Corporation and P5,936.57 from S.S.S. Group of Accredited Insurers but Travellers
Multi-Indemnity refused so it demanded the balance from the other three but they refused so they filed
 Sec. 18. No contract or policy of insurance on property shall be enforceable except for the benefit against them
of some person having an insurable interest in the property insured  Insurance Commission, CFI: absolved Travellers on the basis that Arsenio Cua was claiming and
 A non-life insurance policy such as the fire insurance policy taken by petitioner-spouses over their NOT Tai Tong Chuache
merchandise is primarily a contract of indemnity. Insurable interest in the property insured must exist  Palomo Appealed
a t the time the insurance takes effect and at the time the loss occurs. The basis of such requirement of  Travellers reasoned that the policy is endorsed to Arsenio Chua, mortgage creditor
insurable interest in property insured is based on sound public policy: to prevent a person from taking  Tai Tong Chuache & Co. filed a complaint in intervention claiming the proceeds of the
out an insurance policy on property upon which he has no insurable interest and collecting the fire Insurance Policy issued by travellers
proceeds of said policy in case of loss of the property. In such a case, the contract of insurance is a  affirmative defense of lack of insurable interest that before the occurrence of the peril
mere wager which is void under Section 25 of the Insurance Code. insured against the Palomos had already paid their credit due the petitioner
 SECTION 25. Every stipulation in a policy of Insurance for the payment of loss, whether the ISSUE: W/N Tai Tong Chuache & Co. has insurable interest
person insured has or has not any interest in the property insured, or that the policy shall be received as
proof of such interest, and every policy executed by way of gaming or wagering, is void
 Section 17. The measure of an insurable interest in property is the extent to which the insured HELD: YES. Travellers Multi-Indemnity Corporation to pay Tai Tong Chuache & Co.
might be damnified by loss of injury thereof
Page 35 of 39
 when the creditor is in possession of the document of credit, he need not prove non-payment for it  section 55:
is presumed  the mere transfer of a thing insured does not transfer the policy, but suspends it until the
 The validity of the insurance policy taken b petitioner was not assailed by private same person becomes the owner of both the policy and the thing insured
respondent. Moreover, petitioner's claim that the loan extended to the Palomos has not yet been paid  Undoubtedly these policies of insurance might have been so framed as to have been "payable to
was corroborated by Azucena Palomo who testified that they are still indebted to herein petitioner the San Miguel Brewery, mortgagee, as its interest may appear, remainder to whomsoever, during the
 Chua being a partner of petitioner Tai Tong Chuache & Company is an agent of the partnership. continuance of the risk, may become the owner of the interest insured." (Sec 54, Act No. 2427.) Such a
Being an agent, it is understood that he acted for and in behalf of the firm clause would have proved an intention to insure the entire interest in the property, not merely the
 Upon its failure to prove the allegation of lack of insurable interest on the part of the petitioner, insurable interest of the San Miguel Brewery, and would have shown exactly to whom the money, in
Travellers must be held liable case of loss, should be paid. But the policies are not so written.
 The blame for the situation thus created rests, however, with the Brewery rather than with the
San Miguel Brewery V. Law Union And Rock Insurance Co. (1920) insurance companies, and there is nothing in the record to indicate that the insurance companies were
requested to write insurance upon the insurable interest of the owner or intended to make themselves
liable to that extent
G.R. No. L-14300 January 19, 1920  If by inadvertence, accident, or mistake the terms of the contract were not fully set forth in the
policy, the parties are entitled to have it reformed. But to justify the reformation of a contract, the
Lessons Applicable: proof must be of the most satisfactory character, and it must clearly appear that the contract failed to
express the real agreement between the parties
 Mortgagor (Insurance)
 In the case now before us the proof is entirely insufficient to authorize reformation.
 Measure of Insurable Interest (Insurance)
 Effect of Change of Interest in Thing Insured (Insurance)
 Effect of transfer of thing insured (Insurance) Insurance Case Digest: Bachrach V. British American Assurance Co. (1910)
Laws Applicable: sec. 16,sec. 19 (now sec. 20),sec. 50,sec.55 (now sec. 58) of the Insurance Code (all old
law) G.R. No. L-5715 December 20, 1910
FACTS:
Lessons Applicable: Effect of Change of Interest in Thing Insured (Insurance)
 In the contract of mortgage, the owner P.D. Dunn had agreed, at his own expense, to insure the
mortgaged property for its full value and to indorse the policies in such manner as to authorize the Laws Applicable:
Brewery Company to receive the proceeds in case of loss and to retain such part thereof as might be
necessary to satisfy the remainder then due upon the mortgage debt. Instead, however, of effecting the FACTS:
insurance himself Dunn authorized and requested the Brewery Company to procure insurance on the
property in the amount of P15,000 at Dunn's expense.  E. M. Bachrach insured goods belonging to a general furniture store, such as iron and brass
 San Miguel insured the property only as mortgagee. bedsteads, toilet tables, chairs, ice boxes, bureaus, washstands, mirrors, and sea-grass furniture stored
 Dunn sold the propert to Henry Harding. The insurance was not assigned by Dunn to Harding. in the ground floor and first story of house and dwelling with an authorized agent of the British
 When it was destroyed by fire, the two companies settled with San Miguelto the extent of the American Assurance Company
mortgage credit.  British American Assurance Company denied alleging that:
 RTC: Absolved the 2 companies from the difference. Henry Harding is not entitled to the  property covered by the policy to H. W. Peabody & Co. to secure certain indebtedness
difference between the mortgage credit and the face value of the policies. due and owing to said company
 Henry Harding appealed.  interest in certain of the goods covered by the said policy is trasnferred to Macke to
ISSUE: secure certain obligations assumed by Macke and on behalf of Bachrach
1. W/N San Miguel has insurable interest as mortgagor only to the extent of the mortgage credit - YES  willfully placed a gasoline can containing 10 gallons of gasoline close to the insured
2. W/N Harding has insurable interest as owner - NO goods
HELD: affirmed  made no proof of the loss with the time required by the condition
 RTC: British American Assurance Company liable to bACHRACH
 section 19 of the Insurance Act:
ISSUE: W/N Bachrach can claim
 a change of interest in any part of a thing insured unaccompanied by a corresponding
change of interest in the insurance, suspends the insurance to an equivalent extent, until the interest in
the thing and the interest in the insurance are vested in the same person
Page 36 of 39
The trial court held the policy null and void, hence this appeal.
HELD: YES. lower court affirmed
Issue: WON Argente and Ocampo were guilty of concealment and thereby misled the insurer into accepting
 keeping of inflammable oils on the premises, though prohibited by the policy, does not void it if the risk?
such keeping is incidental to the business Held: Yes. Petition dismissed.
 It may be added that there was no provision in the policy prohibiting the keeping of paints and
varnishes upon the premises where the insured property was stored. If the company intended to rely Ratio:
upon a condition of that character, it ought to have been plainly expressed in the policy.
 alienation clause - forfeiture if the interest in the property pass from the insured Vicenta de Ocampo, in response to the question asked by the medical examiner, answered no to "Have you
 there is no alienation within the meaning of the insurance law until the mortgage acquires a right ever consulted a physician for or have you ever suffered from any ailment or disease of the brain or nervous
to take possession by default under the terms of the mortgage. No such right is claimed to have system?" She also answered “none” as to the question whether she consumed alcohol of not.
accrued in the case at bar, and the alienation clause is therefore inapplicable. To the question, "What physician or physicians, if any, not named above, have you consulted or been
 we can not find that there is a preponderance of evidence showing that the plaintiff did actually set treated by, within the last five years and for what illness or ailment?" she answered "None."
fire or cause fire to be set to the goods in question
 It does not positively appear of record that the automobile in question was not included in the But the facts show that she was taken to San Lazaro Hospital, her case was diagnosed by the admitting
other policies. It does appear that the automobile was saved and was considered as a part of the physician as "alcoholism”, moreover, she was diagnosed with "phycho-neurosis."
salvaged. It is alleged that the salvage amounted to P4,000, including the automobile. This amount
(P4,000) was distributed among the different insurers and the amount of their responsibility was Section 25 of the Insurance Code defined concealment as "a neglect to communicate that which a party
proportionately reduced. The defendant and appellant in the present case made no objection at any knows and ought to communicate."
time in the lower court to that distribution of the salvage. The claim is now made for the first time.
The court held that the alleged concealment was not immaterial and insufficient to avoid the policy. In an
action on a life insurance policy where the evidence conclusively shows that the answers to questions
Argente v West Coast G.R. No. L-24899 March 19, 1928 concerning diseases were untrue, the truth of falsity of the answers become the determining factor. If the
J. Malcolm true facts been disclosed by the assured, the insurance would never have been granted.

Facts: Concealment must, in the absence of inquiries, be not only material, but fraudulent, or the fact must have
been intentionally withheld. If no inquiries are made and no fraud or design to conceal enters into the
Bernardo Argente signed an application for joint insurance with his wife in the sum of P2,000. The wife, concealment the contract is not avoided.
Vicenta de Ocampo, signed for the same. All the information contained in the applications was furnished
The assurer is entitled to know every material fact of which the assured has exclusive or peculiar
the agent by Bernardo Argente.
knowledge, as well as all material facts which directly tend to increase the hazard or risk which are known
Argente was examined by Dr. Sta. Ana, a medical examiner for the West Coast. The result was recorded in
by the assured, or which ought to be or are presumed to be known by him. And a concealment of such facts
the Medical Examiner's Report, and with the exception of the signature of Bernardo Argente, was in the vitiates the policy.
hand-writing of Doctor Sta. Ana. But the information or answers to the questions contained on the face of
the Medical Examiner's Report were furnished the doctor by Argente. If the assured has exclusive knowledge of material facts, he should fully and fairly disclose the same,
Vicenta de Ocampo, wife of the plaintiff, was examined at her residence by the same doctor. whether he believes them material or not. The determination of the point whether there has or has not been
The spouses submitted to West Coast Life an amended application, increasing the amount to P15,000, and a material concealment must rest largely in all cases upon the exact terms of the contract.
asked that the policy be dated May 15, 1925. The amended application was accompanied by the documents
Great Pacific v CA G.R. No. L-31845 April 30, 1979
entitled "Short Form Medical Report." In both of these documents appear certain questions and answers.
J. De Castro
A temporary policy for P15,000 was issued to Bernardo Argente and his wife as of May 15, but it was not
delivered until the first quarterly premium on the policy was paid. More than thirty days had elapsed since
the applicants were examined. Each of them was required to file a certificate of health before the policy
was delivered. Facts:
Vicenta de Ocampo died of cerebral apoplexy. Argente presented a claim in due form to the West Coast Ngo Hing filed an application with the Great Pacific for a twenty-year endowment policy in the amount of
Life Insurance Co. for the payment of the sum of P15,000. It was apparently disclosed that the answers P50,000.00 on the life of his one-year old daughter Helen. He supplied the essential data which petitioner
given by the insured in their medical examinations with regard to their health were untrue. West Mondragon, the Branch Manager, wrote on the form. The latter paid the annual premium the sum of
Coastrefused to pay the claim and wrote Argente to the effect that the claim was rejected due to fraud.
Page 37 of 39
P1,077.75 going over to the Company, but he retained the amount of P1,317.00 as his commission for The deposit paid by private respondent shall have to be refunded by Pacific Life.
being a duly authorized agent of Pacific Life.
2. Ngo Hing had deliberately concealed the state of health of his daughter Helen Go. When he supplied
Upon the payment of the insurance premium, the binding deposit receipt was issued Ngo Hing. Likewise, data, he was fully aware that his one-year old daughter is typically a mongoloid child. He withheld the fact
petitioner Mondragon handwrote at the bottom of the back page of the application form his strong material to the risk insured.
recommendation for the approval of the insurance application. Then Mondragon received a letter from
Pacific Life disapproving the insurance application. The letter stated that the said life insurance application “The contract of insurance is one of perfect good faith uberrima fides meaning good faith, absolute and
for 20-year endowment plan is not available for minors below seven years old, but Pacific Life can perfect candor or openness and honesty; the absence of any concealment or demotion, however slight.”
consider the same under the Juvenile Triple Action Plan, and advised that if the offer is acceptable, the The concealment entitles the insurer to rescind the contract of insurance.
Juvenile Non-Medical Declaration be sent to the company.
WEEK 7
The non-acceptance of the insurance plan by Pacific Life was allegedly not communicated by petitioner
Mondragon to private respondent Ngo Hing. Instead, on May 6, 1957, Mondragon wrote back Pacific Life
again strongly recommending the approval of the 20-year endowment insurance plan to children, pointing
out that since the customers were asking for such coverage.
Helen Go died of influenza. Ngo Hing sought the payment of the proceeds of the insurance, but having Saturnino v. Philamlife - False Representation
failed in his effort, he filed the action for the recovery before the Court of First Instance of Cebu, which
ruled against him.
Facts:

> 2 months prior to the insurance of the policy, Saturnino was operated on for cancer, involving complete
Issues: removal of the right breast, including the pectoral muscles and the glands, found in the right armpit.
> Notwithstanding the fact of her operation, Saturnino did not make a disclosure thereof in her application
1. Whether the binding deposit receipt constituted a temporary contract of the life insurance in question for insurance.
> She stated therein that she did not have, nor had she ever had, among others listed in the application,
2. Whether Ngo Hing concealed the state of health and physical condition of Helen Go, which rendered
cancer or other tumors; that she had not consulted any physician, undergone any operation or suffered any
void the policy
injury within the preceding 5 years.
> She also stated that she had never been treated for, nor did she ever have any illness or disease peculiar
to her sex, particularly of the breast, ovaries, uterus and menstrual disorders.
Held: No. Yes. Petition dismissed. > The application also recited that the declarations of Saturnino constituted a further basis for the issuance
of the policy.
Ratio:
Issue:
The receipt was intended to be merely a provisional insurance contract. Its perfection was subject to
Whether or not the insured made such false representation of material facts as to avoid the policy.
compliance of the following conditions: (1) that the company shall be satisfied that the applicant was
insurable on standard rates; (2) that if the company does not accept the application and offers to issue a Held:
policy for a different plan, the insurance contract shall not be binding until the applicant accepts the policy YES.
offered; otherwise, the deposit shall be refunded; and (3) that if the company disapproves the application, There can be no dispute that the information given by her in the application for insurance was false,
the insurance applied for shall not be in force at any time, and the premium paid shall be returned to the namely, that she never had cancer or tumors or consulted any physician or undergone any operation within
applicant. the preceding period of 5 years.
The receipt is merely an acknowledgment that the latter's branch office had received from the applicant the
insurance premium and had accepted the application subject for processing by the insurance company. The question to determine is: Are the facts then falsely represented material? The Insurance Law provides
There was still approval or rejection the same on the basis of whether or not the applicant is "insurable on that “materiality is to be determined not by the event, but solely by the probable and reasonable influence
standard rates." Since Pacific Life disapproved the insurance application of respondent Ngo Hing, the of the facts upon the party to whom the communication is due, in forming his estimate of the proposed
binding deposit receipt in question had never become in force at any time. The binding deposit receipt is contract, or making his inquiries.
conditional and does not insure outright. This was held in Lim v Sun.

Page 38 of 39
The contention of appellants is that the facts subject of the representation were not material in view of the
non-medical nature of the insurance applied for, which does away with the usual requirement of medical
examination before the policy is issued. The contention is without merit. If anything, the waiver of
medical examination renders even more material the information required of the applicant concerning
previous condition of health and diseases suffered, for such information necessarily constitutes an
important factor which the insurer takes into consideration in deciding whether to issue the policy or not.
Appellants also contend that there was no fraudulent concealment of the truth inasmuch as the insured
herself did not know, since her doctor never told her, that the disease for which she had been operated on
was cancer. In the first place, concealment of the fact of the operation itself was fraudulent, as there could
not have been any mistake about it, no matter what the ailment.
Secondly, in order to avoid a policy, it is not necessary to show actual fraud on the part of the insured. In
this jurisdiction, concealment, whether intentional or unintentional entitled the insurer to rescind the
contract of insurance, concealment being defined as “negligence to communicate that which a party knows
and ought to communicate.” The basis of the rule vitiating the contract in cases of concealment is that it
misleads or deceives the insurer into accepting the risk, or accepting it at a rate of premium agreed upon.
The insurer, relying upon the belief that the insured will disclose every material fact within his actual or
presumed knowledge, is misled into a belief that the circumstances withheld does not exist, and he is
thereby induced to estimate the risk upon a false basis that it does not exist.

Page 39 of 39

S-ar putea să vă placă și